You are on page 1of 94

Axis Bank Aptitude-Reasoning 4388

Axis Bank Sample Paper


Reaoning Ability
1 .Direction : Answer these questions referring to the symbol-letter-number sequence given
below :
N8B@5AR(36YL3AF$4G?TV9yaH7SJ1D
Four of the following five are similar in respect of their positions in the above sequence and
hence form a group. Which one does not belong to the group ?
A. GVoc
B. YAG
C. @pL
D. $Ty
2 . Direction : Answer these questions referring to the symbol-letter-number sequence
given below :
N8B@5AR(36YL3AF$4G?TV9yaH7SJ1D
How many symbol in the sequence are either immediately followed by a letter belonging to
the first half of the English alphabet or immediately followed by a number ?
A. 5
B. 6
C. 7
D. 8
3 . Direction : Answer these questions referring to the symbol-letter-number sequence
given below :
N8B@5AR(36YL3AF$4G?TV9yaH7SJ1D
If the positions of the letters in the sequence are reoccupied by the letters theselves after
getting arranged alphabetically from left, which of the following will indicate the position of
'J' in the arrangement ?
A. Between G and
B. Between 4 and
C. 15th from the right
D. 7th to the left or R
4 . Direction : Answer these questions referring to the symbol-letter-number sequence
given below :
N8B@5AR(36YL3AF$4G?TV9yaH7SJ1D
If the position of the letters in the sequence are reccupied by the letters themselves after
getting arranged alphabeticallyfrom left, Which of the following will not change its position
after the change ?
A. Only N
B. Only R and N
C. Only F
D. None of these

5 . Direction : Answer these questions referring to the symbol-letter-number sequence


given below :
N8B@5AR(36YL3AF$4G?TV9yaH7SJ1D
What is the total number of 'the numbers immediately followed by a letter' and 'the symbols
immediately preceded by a letter' together in the given sequence ?
A. 10
B. 11
C. 9
D. 12
6 . In a certa|n code language the word BREAKDOWN is written as DQGCJFQVP. How will
the word MENSTRUAL be written1 in that code language ?
A. ODPRVQWZN
B. ODPUSTWZN
C. OPDUSTWZN
D. OPDUSWTZN
7 . Four of the following five are similar in relation to their positions in the English alphabet
and hence form a group. Which one does not belong to that group ?
A. PKSOV
B. HPKTN
C. ARDUG
D. JTMXP
8 . If each alphabet is assigned a sequential numerical value in terms of even numbers on
the basis of their position in the English alphabet, viz. A = 2, B = 4,C = 6 and so on, what
will be the value of the word LOCATION ?
A. 180
B. 182
C. 186
D. 178
9 .Direction : In each question below is given a statement followed by two assumptions
numbered I and II. An assumption is something supposed or taken for granted. You have to
consider the statement and the following assumptions and decide which of the assumptions
is implicit in the statement.
Statement: Politicians turning into ministers is a natural transformation even if the quality of
performance rarely matches expectations.
Assumptions: I. No ministers have adequate quality and, skills.
II. Ministers should be able to handle their job efficiently.
A. if only assumption I is implicit.
B. if only assumption II is implicit.
C. if either I or II is implicit.
D. if neither I nor II is implicit.

10 . Direction : In each question below is given a statement followed by two assumptions


numbered I and II. An assumption is something supposed or taken for granted. You have to
consider the statement and the following assumptions and decide which of the assumptions
is implicit in the statement.
Statement: Some ambitious political leaders themselves use grievances of minorities as
empty slogans to grab political power.
Assumptions: I. Such leaders have no mass base nor genuine support, except for an
occasional emotional upsurge.
II. Playing with the minority cards gives some leaders political berth but the status of their
subjects remains unchanged.
A. if only assumption I is implicit.
B. if only assumption II is implicit.
C. if either I or II is implicit.
D. if neither I nor II is implicit.
11 . Direction : Read the following information carefully and answer the questions given
below:
Five Indian players Sachin, Bhutia, Leandef, Anand and Gopichand are related with the
advertisement of different products of different companies Nike, Reebok, Pepsi, Coca Cola
and Adidas , though not respectively . Sachin advertises neither for Sportswear nor for
Rackets. Leander advertises for Shoes but not for the company Coca Cola. Pepsi and Adidas
produce neither Diet Coke nor Sportswear. Gopichand advertises for the company Nike but
neither for Mineral Water nor for Diet Coke. Bhutia and Anand advertise for Adidas and
Pepsi, though not necessarily respectively. Coca Cola does not produce Mineral Water.
For Which product does Gopichand advertise ?
A. Rackets
B. Sportswear
C. Rackets or Sportswear
D. Data inadequate
12 . Direction : In the following questions, the symbols ?, ?, ?, ? and ? are used with the
following meaning:
P
P
P
P
P

?
?
?
?
?

Q
Q
Q
Q
Q

means
menas
means
means
means

P
P
P
P
P

is
is
is
is
is

not smaller than Q.


neither greater than nor smaller than Q
not greater than Q
neither smaller than nor equal to Q.
neither greater than nor equal to Q.

Now in each of the following questions. assuming the given statements to be true, find
which of the two conclusions l all ll given below them is/are definetely true.
Statements : M ? N, ll ? Q, Q ? M

Conclusions : l. ll ? M ll. Q ? N
A. if only conclusion l is true
B. if only conclusion ll is true
C. if only conclusion l or ll is true
D. if neither l nor ll is true
13 . Direction : In the following questions, the symbols ?, ?, ?, ? and ? are used with the
following meaning:
P ? Q means P is not smaller than Q.
P ? Q menas P is neither greater than nor smaller than Q
P ? Q means P is not greater than Q
P ? Q means P is neither smaller than nor equal to Q.
P ? Q means P is neither greater than nor equal to Q.
Now in each of the following questions. assuming the given statements to be true, find
which of the two conclusions l all ll given below them is/are definetely true.
Statements : C ? B, l ? S, S ? C
Conclusions : l. B ? S ll. C ? L
A. if only conclusion l is true
B. if only conclusion ll is true
C. if only conclusion l or ll is true
D. if neither l nor ll is true
14 . Direction : In the following questions, the symbols ?, ?, ?, ? and ? are used with the
following meaning:
P ? Q means P is not smaller than Q.
P ? Q menas P is neither greater than nor smaller than Q
P ? Q means P is not greater than Q
P ? Q means P is neither smaller than nor equal to Q.
P ? Q means P is neither greater than nor equal to Q.
Now in each of the following questions. assuming the given statements to be true, find
which of the two conclusions l all ll given below them is/are definetely true.
Statements : l ? H, E ? F, l ? F
Conclusions : l. E ? l ll. H ? E
A. if only conclusion l is true
B. if only conclusion ll is true
C. if only conclusion l or ll is true or if neither l nor ll is true
D. if both l and ll are true.
15 . Direction : In the following questions, the symbols ?, ?, ?, ? and ? are used with the
following meaning:
P ? Q means P is not smaller than Q.
P ? Q menas P is neither greater than nor smaller than Q
P ? Q means P is not greater than Q
P ? Q means P is neither smaller than nor equal to Q.

P ? Q means P is neither greater than nor equal to Q.


Now in each of the following questions. assuming the given statements to be true, find
which of the two conclusions l all ll given below them is/are definetely true.
Statements : V ? O, R ? V, O ? B
Conclusions : l. R ? B ll. R ? B
A. if only conclusion l is true
B. if only conclusion ll is true
C. if only conclusion l or ll is true
D. if neither l nor ll is true
16 . Direction : In the following questions, the symbols ?, ?, ?, ? and ? are used with the
following meaning:
P ? Q means P is not smaller than Q.
P ? Q menas P is neither greater than nor smaller than Q
P ? Q means P is not greater than Q
P ? Q means P is neither smaller than nor equal to Q.
P ? Q means P is neither greater than nor equal to Q.
Now in each of the following questions. assuming the given statements to be true, find
which of the two conclusions l all ll given below them is/are definetely true.
Statements : L ? U, T ? L, U ? W
Conclusions : l. T ? W ll. U ? W
A. if only conclusion l is true
B. if only conclusion ll is true
C. if only conclusion l or ll is true
D. if neither l nor ll is true
17 . Direction : In each question below, there are three statements followed by four
conclusions numbered I, II, HI and IV. You have to take the given statements to be true
even if they seem to be at variance with commonly known facts and then decide which of
the given conclusions logically follow(s) from the given statements.
Statements: a. Some chairs are tables.
b. All tables are keys.
c. All locks are keys.
Conclusions: I. Some tables are not chairs.
II. All keys are chairs.
III. Some locks are tables.
IV. Some locks are chairs.
A. Only I follows
B. Only II follows
C. Only III follows
D. None of these
18 . Direction : In each question below, there are three statements followed by four
conclusions numbered I, II, HI and IV. You have to take the given statements to be true

even if they seem to be at variance with commonly known facts and then decide which of
the given conclusions logically follow(s) from the given statements.
Statements: a. All cups are books.
b. Some books are bikes,
c. No bike is a scooter.
Conclusions: l. Some books are cups.
II. Some bikes are cups.
III. Some cups are scooters.
IV. No cup is a scooter.
A. Only I follows
B. Only I and III follow
C. Only I and either III or IV follow
D. Either III or IV follows
19 . Direction : In each question below, there are three statements followed by four
conclusions numbered I, II, HI and IV. You have to take the given statements to be true
even if they seem to be at variance with commonly known facts and then decide which of
the given conclusions logically follow(s) from the given statements.
Statements: a. All stands are papers.
b. Some pins are papers.
c. All pins are phones.
Conclusions: I. Some phones are not papers.
II. Some stands are phones.
III. Some pins are stands.
IV. Some phones are not stands.
A. Only either II or III follows
B. Only I and II follow
C. Only either II or IV follows
D. Only I and II or IV follow
20 . Direction : In each question below, there are three statements followed by four
conclusions numbered I, II, HI and IV. You have to take the given statements to be true
even if they seem to be at variance with commonly known facts and then decide which of
the given conclusions logically follow(s) from the given statements.
Statements: a. No page is a copy.
b. Some cards are not rectangles.
c. Some copies are rectangles.
Conclusions: I. Some pages are not rectangles.
II. No rectangles are pages.
III. Some copies are not pages.
IV. Some rectangles are not pages.
A. Only either II or IV follows
B. Only IV follows
C. Only either I or II follows
D. Only III and IV follow

31 . Direction (Q. 1-5) : Read the following information and answer the questions that
follow.
P, Q, R, S, T, U, V and W are eight friends. There of them play Hockey and Tennis each two
of them play Gold. They all are of different heights. The shortest does not play Hockey and
the tallest does not play Gold. U is taller than P and S, but shorter than Q and W. T, who
does not play Hockey, is taller than Q and is second tallest. V is shorter than S, but taller
than P. W plays Tennis with S, and is fourth from the top. V does not play either Hockey or
Gold. Q does not play Gold.
Who is the shortest ?
A. V
B. S
C. P
D. Data inadequate
32 . Who is the tallest ?
A. Q
B. W
C. R
D. Data inadequate
33 . Which of the following pairs of friends plays Golf ?
A. TU
B. TW
C. WU
D. None of these
34 . What is U's position from the top, when they are arranged in descending order of their
height ?
A. Fifth
B. Fourth
C. Sixth
D. Can't be determined
35 . Which of the following groups of friends plays Hockey ?
A. RTP
B. RQU
C. RQP
D. RPU
36 . Direction (Q. 6-10) : Read the following in information to answer the questions:
'P + Q' means P is the father of Q.
'P - Q' means P is the sister of Q.
'P x Q' means P is the husband of Q.
'P Q' means P is the wife of Q.

Which of the following menas P is the mother-in-law of R ?


A. P V + T - W x R - S
B. P x M + N x R - S
C. P - S L + M x R
D. V x P - S L + M - R
37 . What should come in place of question mark (?) in the given expression to make 'X is
the great-aunt of Z' definitely true ?
X-WV+Y?L+M-Z
A. +
B. X
C. D. Either - or
38 . Which of the following means 'C is the niece of R' ?
A. R + P - S + X + C - L
B. P - R + Q x S - C - N
C. B x R - P Q + C - M
D. R x S - Q - P M x C
39 . If P Q + R - C -s- W + L is true, which of the following is true ?
A. L is the niece of R.
B. Q is the maternal uncle of L.
C. P is the paternal great-aunt of L.
D. Q is the maternal grandfather of L.
40 . In a coded language "come at once mother very sick" is written as "XLNV ZG LMXV
NLGSVI EVIB WJXP". What is the code for 'sister' ?
A. WJWMXI
B. WJWGVI
C. WJWSVI
D. WJWLVI
Axis Bank Placement Paper 4490
Axis Bank Placement Paper
1. An account holder draws a cheque on a?
(1) Banker of the Payee
(2) Banker of the Drawee
(3) Banker of the Drawer
(4) Banker of the Endorsee
(5) None of the above
2. The Bank should comply and intimate the compliance of Award to Ombudsman?
(1) Within Two Months
(2) Within 3 months

(3) Within one month


(4) Within one year
(5) None
3. Capital adequacy is worked out based on?
(1) Total demand and time liabilities
(2) Net demand and time assets
(3) Risk weighted assets
(4) Risk weighted liabilities
(5) None of the above
4. Statutory Liquidity Ratio (SLR) can be maintained in the following ways?
(1) Cash
(2) Gold
(3) Unencumbered approved securities
(4) All of these
(5) None of the above
5. What was the deposits criteria of 14 Banks nationalized on 19th July 1969?
(1) Rs.1000 Crore
(2) Rs. 500 Crore
(3) Rs. 100 Crore
(4) Rs. 50 Crore
(5) None of the above
6. What are the features of selective credit control?
(1) They distinguish between essential and non-essential uses of Bank credit
(2) Only non-essential uses are brought under the scope of Credit Bank controls
(3) They effect not only the lenders but also the borrowers
(4) All the above
(5) None of the above
7. Which of the following are the targets for different categories of priority sector?
(1) Overall target 40% of net Bank credit
(2) Agriculture loans (direct and indirect) 18%
(3) Priority sector target for foreign Banks is 32%
(4) All the above
(5) None of the above
8. Benefits in core Banking is ?
(1) Reliable centralized data recovery
(2) Data warehousing and data mining technologies
(3) Integrated customer centric services
(4) All the above
(5)None

9. Cheque truncation can be done by?


(1) Using MICR data
(2) Sending cheque by speed post
(3) Using image processing
(4) 1 & 3
(5) None of the above
10. Bank rate is decided by?
(1) Indian bank association
(2) Individual commercial bank
(3) Finance ministry
(4) Any one of the above can decide
(5) None of the above
11. What is nomination?
(1) A facility given to depositors by the Government
(2) It confers a right to the nominee to receive payment of a deposit after death of
depositors
(3) It is a process of appointing legal heir by the depositors
(4) It is a part of will on behalf of a depositor
(5) None of the above
12. BFS- Board for Financial Supervision constituted in 1994 by RBI to undertake
consolidated supervision of?
(1) Commercial Banks
(2) Financial Institutions
(3) Non Banking Finance Companies
(4) All the above
(5) None of the above
13. 'One Family one Bank' is the tagline of which Bank?
(1) Bank of India
(2) Indian Bank
(3) Bank of Maharashtra
(4) Union Bank of India
(5) None of the above
14. Research Institutions set up by Reserve Bank of India are?
(1) IDRBT (Institute for Development and Research in Ban- king Technology), Hyderabad
(2) IGIDR(Indira Gandhi Institute of Development Research), Mumbai
(3) NIBM (National Institute of Bank Management), Pune
(4) Only a & b
(5) All the above
15. Who is the Chairman of the Technical Committee formed to make a study on
improvement in Mobile Banking:

(1)
(2)
(3)
(4)
(5)

Bimal Jalan
Rahuram Rajan
B Sambamurthy
Pulak Kumar Sinha
None of the above

16. In NEFT E stands for


(1) Election
(2) Email
(3) Enlisted
(4) Ecstasy
(5) Electronic
17. The Basic Savings Bank Deposit Account (BSBDA) issued by banks is offered to which of
the following customers?
1) Farmers
2) Daily wage earners
3) Self Help Group members
4) Students
5) All the customers
18. What is the validity of Kisan Credit Card?
(1) 1 year
(2) 2 year
(3) 10 years
(4) 5 years
(5) 3 years
19. The Negotiable instrument is _____.
(1) Cheque
(2) Demand Draft
(3) Bill of Exchange
(4) All of the above
(5) Deposit Slip
20. The minimum paid up capital required for both small and payment banks is
(1) Rs 500 Crore
(2) Rs 50 Crore
(3) Rs 100 crores
(4) Rs 200 Crore
(5) None of these
21.
(A)
(B)
(C)

Bring out the only incorrect statement:


Reserve Repo operation by RBI aims are injecting/increasing liquidity
SDR refers to special drawing
Rupee appreciation results in decrease in imports

(D) Increase in inflation rate leads to decline in real interest rate


22. What is a Repo Rate?
(A) It is a rate at which RBI sell government securities to banks
(B) It is a rate at which banks borrow rupees from RBI
(C) It is a rate at which RBI allows small loans in the market
(D) It is a rate which is offered by Banks to their most valued customers or prime customers
(E) None of these
23. Which of the following cannot be called as a Debt Instrument as referred in financial
transactions?
(A) Certificate of Deposits
(B) Bonds
(C) Stocks
(D) Commercial Papers
(E) Loans
24. Whenever RBI does some Open Market Operation Transaction, actually it wishes to
regulate which of the following?
(A) Inflation only
(B) liquidity in economy
(C) Borrowing powers of the banks
(D) Flow of Foreign Direct Investments
(E) None of these
25. The maximum amount of the total Revenue earned by the government of India comes
from:
(A) Income Tax
(B) Customs Duty
(C) Excise Duty
(D) Value Added Tax
(E) Corporate Tax
26. In economics it is generally believed that the main objective of a Public Sector Financial
Company like Bank is to:
(A) Employ more and more people
(B) Maximize total profits
(C) Maximize total production
(D) Provide financial service to the people of the nation of its origin across the country
(E) Sell the goods at subsidized cost
27. Capital Market Regulator is
(A) RBI
(B) IRDA
(C) NSE
(D) BSE

(E) SEBI
28. Which is the first Indian company to be listed in NASDAQ ?
(A) Reliance
(B) TCS
(C) HCL
(D) Infosys
(E) None of these
29. What is Call Money ?
(A) Money borrowed or lent for a day or over night
(B) Money borrowed for more than one day but upto 3 days
(C) Money borrowed for more than one day but upto 7 days
(D) Money borrowed for more than one day but upto 14 days
(E) None of these
30. The Bank of Calcutta, Bank of Bombay and Bank of Madras were merged in 1921 to
form
(A) Reserve Bank of India
(B) Imperial Bank of India
(C) Bank of India
(D) Union Bank of
(E) None of these
Axis Bank Interview Questions 4372
Axis Bank Interview Questions
* Introduce yourself
* Tell me your strengths and weakness
* Why Banking?
* Why do you want to join Axis bank ?
* What is difference between nominal and real account ?
* What is the concept of finance accounting?
* What is explosure ?
* How to reduce axis bank emi ?
* What is the e mail id of customercare of axis bank powerkome loan ?
* What is accounting for bank accounts ?

* Why sales is required ?


* What challenges are you looking for in a position?
IBPS Bank PO (CWE) Quantitative Aptitude Question Paper 9730
IBPS Bank PO CWE QUANTITATIVE APTITUTE Questions
Directions (Qs. 1-20 ) : What should come in place of the question mark (?) in the following
questions ?
1. (-251 X 21 X -21) ? = 158.13
A. 250 B. 400 C.300 D.150

2. 25.6 % of 250 + ? = 119


A. 4225 B. 3025 C.2025 D. 5625 E. None of these

3. 36865 + 12473 + 21045 - 44102 = ?


A. 114485 B. 28081 C. 26281 D.114845 E. None of these

4. (15.20)2

- 103.04 ) ? = 8

A. 12 B. 6.5 C.8.2 D.16 E. None of these

5. 0.4 X 1.5 / ? 0.2


A. 2 B. 3 C. 1.2 D. 0.3 E. None of these

6. 7428 X 3/4 X 2/9 X ? = 619


A. 0.5 B. 1.5 C. 0.2 D. 2.4 E. None of these

7. (560 ) 32 ) X (720 48) = ?


A. 262.5 B. 255 C. 263.5 D. 271.25 E.None of these

8. 748 X ? X 9 = 861696
A. 122 B. 132 C. 128 D. 124 E. None of these

9. 3.2 % of 500 X 2.4 % of ? = 288


A. 650 B. 700 C. 600 D.750 E. None of these

10. 333 X 33 X 3 = ?
A. 32697 B. 36297 C. 32679 D. 39267 E. None of these

11. (36) 2 + (47) 2 ? = 28.04


A. 55 B. 75 C.105 D. 125 E. None of these

12. 3? = (28 X 24 ) 14
A. 85184 B. 140608 C.97336 D. 117649 E. None of these

13. 14785 358 4158 9514 = ?


A.755 B. 825 C.721 D. 785 E. None of these

14. 156 + 328 X 0.8 = ?


A. 287.2 B. 352.8 C. 418.4 D. 451.2 E. None of these

15. 1148 28 X 1408 32 = ?


A. 1800 B. 1804 C. 1814 D. 1822 E. None of these

16. 3148877 = ?
A. 51 B. 59 C.53 D. 49 E. None of these

17. 1425 + 8560 + 1680 200 = ?


A. 58.325 B. 9973.4 C. 56.425 D. 9939.4 E. None of these

18. [(12) 2 X (14) 2 } (16) = ?


A. A.282.24 B.1764 C.126 D. 104.25 E. None of these

19. 2 & 4/25 + 3 & 1/5 = ?


A. 1 &7/15 B. 6 & 4/15 C. 5 & 4/15 D. 5 & 9/25 E. None of these

20. ? X 3025 = 2695


A. 2401 B. 2209 C. 2601 D. 2304 E. None of these

Directions (21-25) : What should come in place of the question mark (?) in the following
number series ?
21. 7413 7422 7440 ? 7503 7548
A. 7464 B. 7456 C. 7466 D.7466 E. None of these

22. 4 16 36 64 100 ?
A. 120 B. 180 C. 136 D. 144 E. None of these

23. 12 33 96 ? 852 2553


A.285 B. 288 C.250 D.540 E. None of these

24. 70000 14000 28000 ? 112 22.4


A. 640 B. 420 C.560 D. 540 E. None of these

25. 102 99 104 97 106 ?


A. 96 B. 95 C. 100 D.94 E. None of these

26. If the numerator of a fraction is increased by 200% and the denominator of the
fraction is increased by 120% , the resultant fraction is 4/11. What is the original fraction ?
A. 4/15 B. 3/11 C.5/12 D. 6/11 E. None of these

27. Mr. Rao invests a sum of Rs.41.250 at the rate of 6+ p.c.p.a. What approximate amount
of compound interest will he obatain at the end of 3 years ?
A. Rs.8,100 B. Rs. 7,425 C. Rs. 8,210 D. Rs.7,879 E. Rs. 7,295

28. The profit after selling a pair of trousers for Rs.863 is the same as the loss incurred after
selling the same pair of trousers for Rs.631. What is the cost price of the pair of trousers ?
A. Rs. 750 B. Rs.800 C. Rs. 763

D. Cannot be determined E. None of these

29. The ration of the length and the breadth of a rectangular plot is 6:5 respectively ; if the
breadth of the plot is 34 metres less than the length, what is the perimeter of the
rectangular plot ?
A. 374 metres B. 408 metres C. 814 metres D.748 metres E. None of these

30. Ninad, Vikas and Manav enter into a partnership. Ninad invests some amount at the
beginning. Vikas invests double the amount after 6 months and Manav invests thrice the
amount invested by Ninad after 8 months. They earn a Profit of Rs.45,000 at the end of the
year. What is Manavs share in the profit ?
A. Rs.25,000 B. Rs.15,000 C. Rs.12,000 D. Rs.9,000 E. None of these

31. The cost of 7 tables and 12 chairs is Rs. 48,250. What is the cost of 21 tables and 36
chairs?
A. Rs. 96,500 B. Rs.1,25,500 C. Rs. 1,44,750 D. Cannot be determined E. None of these
32. A can complete a piece of work in 12 days. A and B together can complete the same
piece of work in 8 days. In how many days can B alone complete the same piece of work ?
A. 15 days B. 18 days C. 24 days D. 28 days E. None of these

33. What approximate value should come in place of the question mark (?) in the following
question ?
6523 544 X 1.2 = ?
A. 21 B. 33 C.14 D. 8 E.28

34. Raman drove from home to a neighbouring town the speed of 50 kms/hr and on his
returning journey, he drove at the speed of 45 kms/hr. and also took an hour longer to
reach home. What distance did he cover each way ?
A. 450 kms. B. 225 kms. C. 900kms. D. 500kms. E. None of these

35. 3/4th of 2/9th of 1/5th of a number is 249.6 What is 50 % of that number ?

A. 3794 B. 3749 C. 3734 D. 3739 E. None of these

36. The average weight of a group of 75 girls was calculated as 47 kgs. It was later
discovered hat the weight of one of the girls was read as 45 kgs., Whereas her actual weight
was 25 klgs. What is the actual average weight of the group of 75 girls ? (Rounded off to
two digits after decimal)
A. 46.73 kgs. B. 46.6 kgs. C. 45.96 kgs. D. Cannot be determined E. None of these
37. In how many different ways can the letters of the word MIRACLEbe arranged?
A. 720 B. 5040 C. 2520 D.40320 E. None of these

38. The sum of the two digits of two digit number is 12 and the difference between the two
digits of the two digit number I s6. What is the two-digit number ?
A. 39 B. 84 C.93 D. Cannot be determined E. None of these
39. If an amount of Rs.74, 336 is equally divided amongst 150 people, how much
approximate amount would each person get ?
A. Rs. 522 B. Rs.485 C. Rs.496 D. Rs.488 E. Rs.510

40. What is 170 % of 1140 ?


A. 1938 B. 1824 C. 1995 D.1881 E. None of these

41. Mr. Sarang invests 6 % of his monthly salary i.e. Rs. 2,100 on insurance policies . Also
he invests 8% of his monthly salary on family medi claim policies and another 9 % of his
salary on NSCs. What is the total annual amount invested by Mr. Sarang ?
A. Rs.11,400 B. Rs. 96,600 C. Rs.8,050 D. Rs.9,500 E. None of these

42. A trader sells 145 metres of cloth for Rs.12,325 at the profit of Rs.10 per metre of cloth.
What is the cost price of one metre of cloth ?
Rs., Rs.65 B. Rs.75 C. Rs.95 D. Rs.85 E. None of these

43. There are 7 dozaen candles kept in a box. If there are 14 such boxes, how many
candles are there in all the boxes together ?
A. 1176 B. 98q C.1216 D. 168 E. None of these

44. Which of the following smallest number should be added to 6659 to make it a perfect
square ?
A. 230 B. 65 C.98 D.56 E. None of these

45. What is the greater of two number whose product is 1092 and the sum of the two
numbers exceeds their difference by 42 ?
A. 48 B. 44 C. 52 D. 54 E. None of these

46. Veena obtained an amount of Rs.8,376 as simple interest on a certain amount at 8


p.c.p.a after 6 years. What is the a mount invested by Veena ?
A. Rs. 17,180 B. Rs.18,110 C. Rs.16,660 D. Rs.17,450 E. None of these

47. Two numbers are such that the sum of twice the first number and thrice the second
number is 36 and the sum of thrice the first number and twice the second number is 39.
Which is the smaller number ?
A.9 B. 5 C. 7 D.3 E. None of these

48. The difference between the present ages of Arun and Deepak is 14 years . Seven years
ago the ratio of their ages was 5 : 7 respectively. What is Deepaks present age ?
A. 49 years B. 42 Years C. 63 years D. 35 years E. None of these

49. A, b,c,d and e are 5 consecutive even numbers. If the sum of a and d is 162, what is
the sum of all the numbers ?
A. 400 B. 380 C. 420 D. Cannot be determined E. None of these

50. What is 348 times of 265 ?


A. 88740 B. 89750 C. 95700 D. 92220 E. None of these

Complete preparation kit for Bank P O and Common Written Examination


Placement Paper 4222

IBPS clerical cadre written test examination questions with answers and
explanations, IBPS aptitude, Reasoning Arithmetic, logical Reasoning questions
with answer
1. Ram went 15km to the west from his house, then he turned left and walked 20 km. He
then turned east and walked 25 km and finally turning left covered 20 kms.How far was he
from his house
5 kms
10kms-Answer
40 kms
80 kms
2. Vishnu ranks 16th from the top and 49th from the bottom in a class . How many students
are there in the class
66
65-Answer
64
63
3. Mahesh is older than Samir and Jayanth.Navin is older than Samir but younger than
Jacob.Jayanth is older than Jacob.Who among them is the oldest
Jayanth
Jacob
Navin
Mahesh-Answer
4. Choose the option related to the given numbers
21, 37, 53
19, 36, 55
23, 39, 55-Answer
35, 62, 79
47, 64, 80
5. Find out the next term in the given series
34, 18, 10, 6, 4, ....
3-Answer
2
1
0

6. DWH : WDS :: FUL : .........


ELV
OFU
UFO-Answer
FOU
7. Pointing to a photograph, a man said, I have no brother or sister but that man's father is
my father's son.Whose photograph was it
His own
His son's-Answer
His father's
His nephew's
8. In an examination 36 % is the pass mark.If an examinee gets 17 marks and fails by 10
marks, what was the maximum mark
70
75-Answer
65
55
9. If 76 is divided into four parts proportional to 7: 5: 3: 4 the smallest part is
12-Answer
15
16
19
10. A train 280 m long, running with a speed of 63 km/hr.will pass an electric pole
in .........seconds
20 sec
16sec-Answer
15 sec
18 sec
11. Articles which are bought for Rs 400 are sold for Rs 560.Find the profit percent
45
60

40
65
12. If the total rainfall in a month is 75cm, then the average daily rainfall is
3.5 cm
2.5 cm-Answer
1.5 cm
4 cm
13. A man sells an article for Rs 330, at a gain of 10 %.What did it cost him
300-Answer
340
330
360
14. 2.65 - 0.5 = ..........
2.60
3.70
3.15
2.15-Answer
15. If 27 x 29 = 783, then 0.27 x 290 = .........
7830
783
78.3-Answer
7.83
16. Which of the following names will be second in an alphabetical list ?
Andhra Pradesh
17

Haryana1. Which of the following names will be second in an alphabetical list ?

Andhra Pradesh
Haryana-Answer
Himachal Pradesh
Kerala

18

. Which of the following names would be fourth in the telephone directory ?

Sita Ram Agrawal-Answer


Seeta Ram Agrawal
Seta Ram Agarwal
Seeta Ram Agarawal
19 . Ina military mode, TARGET is written as RATTEG. Using the same code BRIDGE
should be written as
DGEBRI-Answer
IRBDGE
BRIEGD
IRBEGD
20 . If x means , - means x , means +, and + means -, then the value of the
expression
(3- 15 11) x 8 + 6 will be
1-Answer
0
8
4
21. A number exceeds its one fifth by 20, what is the number
Answer : 25
22. Pointing to Veena in the photograph, Vishnu said, She is the daughter of my
Grandfather's only son.How is Veena related to Vishnu
Answer : Sister
23. If 27 x 29 = 783, then 0.27 x 290 = .........
Answer : 78.3
24. A man sells an article for Rs 330, at a gain of 10 %.What did it cost him
Answer : 300
25. If the total rainfall in a month is 75cm, then the average daily rainfall is
Answer : 2.5 cm
26. Which number should come in the blank space to complete the series

2, 6, 12, 36, 72, ........


Answer : 216
27. Find the value of 7 + 8 = ...........
Answer : 2
28. If the radius of a circle is increased by 6 % , then its area is increased by ..........%
Answer : 12.36 %
29. Which is the number that comes next in the following series.
4, 6, 12, 14, 28, 30, ........
Answer : 60
30. The downstream speed of a boat is 15 kmph. and the speed of the stream is 2
kmph.Then the upstream speed of the boat is ...
Answer : 11 kmph
31. When a number x is divided by 4 we get the cube of another number y.Find x if y3<10.
Answer : 32
32. The smallest number which if divided by 9, 12, and 16 each time 5 remains. The
number is ..........
Answer : 149
33. Some boys are standing in a queue. If the 10th boy from behind is 5 behind the 12th
boy from the front. how many boys are there in the queue
Answer : 26
34. A cube is to be painted. No two adjacent faces can be coloured the same. What is the
least number of colours needed
Answer : 3
35. There are four roads. I have come from the south and want to go to the temple. The
road to the right leads me away from the coffee house, straight ahead it leads only to a
college. In which direction is the temple
Answer : East
36. A , B, C are intelligent.A, D, E are laborious. D, C, E are honest. A, B, E are ambitious.
Which of the following are not honest
Answer : A, B

37. The average age of a husband and wife was 22 years when they were married five years
back. What is the present average age of the family if they have a three year old child
Answer : 19 years
38. A father tells his son, I was your present age when you were born. If father is 36 now,
how old was the son five years back
Answer : 13
39. If 3 + 3 = 27: 4 +7 = 84 and 5 +7 = 105, then 6 + 7= .........
Answer : 126
40. Find out the number that comes in the missing position of the following number series:
0, 7, 26, 63, 124,.....
Answer : 215
41. What will be the next number if the series of numbers is as follows 3, 11, 27, 59,..........
Answer : 123
42. What will be the next number in the following series of numbers 2,4,16,......
Answer : 256
43. Bangladesh : Dhaka ; Austria : ...........
Answer : Vienna
44. Pareekkutty: Madhu; Chemban Kunju : ...........
Answer : Kottarakara Sreedharan Nair
45. Pather Panjali: Sathyajith Ray ;Athithi : .................
Answer : K . P. Kumaran

Free online mock test for IBPS-CWE Clerical 2012 Exam (Just same as IBPS Pattern Fully solved Questions with answer and Explanation)
Try It Now
1 Two students fought a college election. The winning student got 60 % of the total votes
and won the election by 144 votes only. What was the total number of votes polled
Answer : 720

2. 200 20 100 = ........


Answer : 0.1
3. If 75% of a number is added to 75 the result is the number itself, then the number is
Answer : 300
4. 375 25 - 19 = ..........
Answer : -4
5. 2.65 - 0.5 = ..........
Answer : 2.15
6. Articles which are bought for Rs 400 are sold for Rs 560.Find the profit percent
Answer : 40
7. A train 280 m long, running with a speed of 63 km/hr.will pass an electric pole in
.........seconds
Answer : 16sec
8. If 76 is divided into four parts proportional to 7: 5: 3: 4 the smallest part is
Answer : 12
9. In an examination 36 % is the pass mark.If an examinee gets 17 marks and fails by 10
marks, what was the maximum mark
Answer : 75
10. The smallest prime number is
Answer : 2
11. What is the largest number that divides 10 and 15 without a remainder
Answer : 5
12. The ratio between two numbers is 2: 3. Their LCM is 36. What is the smaller number.
Answer : 12
13. The marks obtained by a student in five subjects are 68, 73, 62, 85 and 79. Find the
average score.
Answer : 73.4

14. 21 3 ( 10 - 3 ) - 20 + 1 =
Answer : 30
15. 0.0076 1.9 = ............
Answer : 0.004
16. Which term of the progression 4, 7, 10, .........is 76
Answer : 25
17. When 8 % of 630 is added to 12 % of 315, the sum is
Answer : 88.2
18. One half of the students at an Indian University study German and 1/3 of others study
French.The remaining 600 do not study any foreign language. How many students are there
in the university.
Answer : 1800
19. A 300 m long train passes a pole in 12 secs. What is its speed in km/hr
Answer : 90 km/hr
20. A train runs for 2 hrs at the speed of 35 km/hr. It runs for 3 1/2 hrs at the speed of 60
km/hr and then runs for 2 1/2 hrs at the speed of 70 km/hr. Find the average speed of the
train
Answer : 56.87 km/hr
21. In an examination 40 % students fail in maths, 30 % in English and 15 % in both.Find
the pass percentage
Answer : 45 %
22. A class has R rows of desks with D desks in each row. On a particular day when all the
pupils were present, three seats were found vacant. The number of pupils in the class is
Answer : DR - 3
23. Nalini purchases a cycle for Rs. 660 including sales tax.If the rate of sales tax is 10 %
find out the sale price of the cycle
Answer : Rs 600
24. Which one of the following is having greater value
Answer : 5:3

25. What must be added to the numbers 6, 10, 14 and 22 so that they became successive
proportional
Answer : 2
Placement Paper 10775
IBPS Banking Question papers based on Test of Reasoning-Made Simple -Easy
methods to crack verbal reasoning questions in Bank Test, Verbal Classification
Notes,types of verbal classification Choose the odd word out,Choose the odd pair
out ,Choose the odd number out ,Choose the odd numeral-pair group and Analogy
- Tips and tricks to crack bank test,IBPS CWE analogy questions 'Analogy' means
'correspondence'. Verbal analogical tests use words,Alphabet Analogy,Number
Analogy, Completing the analogous pair

IBPS and Bank PO Clerical -Verbal Classification Questions and Tips :'Classification' means to assort the items of a given group on the basis of a certain
common quality they possess and then spot the odd one out. Different types of questions
that can be asked are as follows :(1) Choose the odd word out : In these questions, you are given a group of certain items
out of which all except one are similar to one another in some manner. You have to identify
the item which does not fit into the given group.
(2) Choose the odd pair out : In these type of questions, you are given certain pairs of
words out of which the words in all the pairs except one bear a certain common relationship.
You have to decipher the relationship and choose the pair in which the words are differently
related than the remaining pairs.
(3) Choose the odd number out : In these type of questions certain numbers are given,
out of which all except one are alike in some manner while one is different and this number
is to be chosen as the answer.
(4) Choose the odd numeral-pair group : In these type of questions, certain pairs or
groups of numbers are given out of which all except one are similar in some manner while
one is different. The numbers in these similar pairs may have the same property or may be
related to each other according to the same rule. You have to choose the odd pair/group.
IBPS and Bank PO Clerical ANALOGY Questions and Tips
'Analogy' means 'correspondence'. Verbal analogical tests use words, letters and digits and
require logical common sense reasoning to solve questions based on these. In these
questions, a particular relationship is given and another similar relationship has to be
identified from the alternatives provided. These analogical questions test the candidate's
overall knowledge and ability to think concisely and accurately.
Different types of questions that can be asked on Analogy are as follows :-

(1) Completing the analogous pair : In this, two words which are related to each other
are given along with a third word. You have to find the relationship between the first two
words and choose the word from the given options that bears the same relationship with the
third word as the first two words.
(2) Choose the analogous pair : In these type of questions, a pair of words is given,
followed by four pairs of words as options. You have to choose the pair in which the words
have the same relationship to each other as the words in the given pair.
(3) Three word Analogy : In these types of questions, a group of three inter-related
words is given. You have to trace the relationship among these three words and choose
another group with similar analogy from the alternatives that are provided.
(1) Number Analogy : Two types of questions are asked on this.
(i) Choosing a similarly related pair as the given number pair on the basis of the relation
between the numbers in each pair.
(ii) Choosing a number similar to a group of numbers on the basis of certain common
properties they possess.
(5) Alphabet Analogy : In these type of questions, two groups of alphabets related to
each other in some way are given. You have to find out his relationship and choose a group
of alphabets that is related in the same way to the third group in the question.

Illustrative Questions
Directions (Q.Nos. 1-5) : - Four of the following five are alike in a certain way and hence
form a group. Which is the one that does not belong to the group ?
1.
(1) Pear
(2) Mango
(3) Orange
(4) Apple
(5) Radish
Ans : Radish - Except radish, all others are fruits. Radish is a modified root.
2.
(1) Sun
(2) Moon
(3) Star
(4) Mars

(5) Universe
Ans : Universe - All except Universe form part of the Universe.
3.
(1) Tailor
(2) Barber
(3) Engineer
(4) Carpenter
(5) Blacksmith
Ans : Barber - All except barber require raw material to work.
4.
(1) X-ray
(2) Radio
(3) Telephone
(4) Computer
(5) Television
Ans : X-ray - All except X-ray are media instruments.
5.
(1) Arc
(2) Diagonal
(3) Tangent
(4) Radius
(5) Diameter
Ans : Diagonal - All except diagonal are terms associated with circle.
Directions (Q.Nos. 1-11) : - Four of the following five are alike in a certain way and
hence form a group. Which is the one that does not belong to the group ?
1.
(1) IJK
(2) ABC
(3) OPQ

(4) EFG
(5) VWX
Ans : VWX - In all the other groups, the first letters is the odd-numbered letter in the
English alphabet.
9
I

10
J

11
K

1
A

2
B

3
C

15
O

16
P

17
Q

5
E

6
F

7
G

But
22
V

23
W

24
X

2.
(1) Tiger
(2) Cow
(3) Bitch
(4) Mare
(5) Lioness
Ans : Tiger

- Except tiger, all others are of feminine gender.

3.
(1) 13
(2) 17
(3) 19
(4) 27
(5) 23
Ans : 27 - Except 27, all others are prime numbers.
4.
(1) Pound
(2) Yen
(3) Ounce
(4) Franc
(5) Dollar
Ans : Ounce - All except ounce are names of currencies of different countries, while ounce
is a unit of weight.

5
(1) Lemon
(2) Orange
(3) Malta
(4) Banana
(5) Pineapple
Ans : Banana

- All except banana are juicy fruits.

6.
(1) Lake
(2) Tank
(3) River
(4) Pool
(5) Pond
Ans : River - All except river contain stagnant water.
7.
(1) Carrot
(2) Guava
(3) Tomato
(4) Pear
(5) Brinjal
Ans : Brinjal - All except brinjal can be eaten raw.
8.
(1) Sit
(2) Hit
(3) Kit
(4) Fit
(5) Knit
Ans : Kit - Except kit, the rest all are verbs. Kit is a noun.
9.

(1) Tab
(2) Pot
(3) Reed
(4) Level
(5) Hold
Ans : Hold - Other words when read backwards give meaningful words.
10.
(1) CFI
(2) UXA
(3) RUX
(4) SVZ
(5) LOR
Ans : SVZ
C

+3

+3
F

+3
X

+3

I
A

+3

+3
X

+3

+3
R

But,
S

+3
V

+4

Therefore, 'SVZ' does not belong to the group.


11.
(1) 117
(2) 74
(3) 153
(4) 108
(5) 99
Ans : 74 - Except 74, all other numbers are completely divisible by 3.

Directions (Q.Nos. 12-16) : - The following questions consist of two words that have a
certain relationship with each other, followed by five pairs of words. Select the pair that has
the same relationship as the original pair of words.
12. Escape : Abscond
(1) Endless : Eternal
(2) Flee : Surrender
(3) Exult : Jubilate
(4) Escalate : Weaken
(5) Confront : Submit
Ans : Endless : Eternal
and eternal.

- Escape and abscond are nearly similar in meaning like endless

13. Host : Hospitable


(1) Artist : Imitative
(2) Guest : Rude
(3) Humanitarian : Altruistic
(4) Idealist : Cynical
(5) Pollyanna : Pessimistic
Ans : Humanitarian : Altruistic - As a host is supposed to be hospitable, in the same way,
a humanitarian is expected to be altruistic.
14. Editor : Magazine
(1) Director : Film
(2) Novel : Writer
(3) Psychiatrist : Neurotic
(4) Librarian : Library
(5) Poem : Poet
Ans : Director : Film - Like all actions of a magazine are interpreted by the editor, in the
same way, all the actions of a film are interpreted by the director.
15. Pain : Misery
(1) Disease : poverty
(2) Despair : loneliness
(3) Ignorance : confusion
(4) Superstition : peasants

(5) Ignore : greet


Ans : Ignorance : Confusion

- Pain causes misery and ignorance causes confusion.

16. ABD : EJL


(1) GFH : IPR
(2) IPR : URT
(3) IPR : OTY
(4) IPR : ORT
(5) GHI : PRT
Ans : IPR : ORT - This is a vowel-consonant based relationship. In the question pair, the
first letters of each term are 'A' and 'E' which are first two vowels (AEIOU). In the same
manner, the first term of the answer pair will begin with 'I' and the second term of the
answer pair will begin with 'O'. In the question pair, we have ABD i.e. the third letter in first
term is obtained by skipping 'C'. In the second term, 'EJL', 'L' is obtained by skipping 'K'. So,
the answer is (4) IPR : ORT.
IBPS clerical aptitude paper 5874

IBPS Bank PO CWE( Common written Examination) for clerical cadre,Probationary


officers, Clerks,IT Specialists officers job recruitment notification, Examination details
,IBPS Regional Rural Bank (RRB) Gramin Bank examination procedure IBPS
examination syllabus, IBPS RRB SBI,SBT...previous years solved question papers IBPS
aptitude, reasoning, Data interpretation, Verbal ability, Computer -IBPS Question Bank
based general awareness question with answers and detailed solutions, IBPS huge
collection of question papers, IBPS free solved sample placement papers, IBPS Tips and
trick for all topics question IBPS latest updatesIBPS clerical cadre recruitment
detials.IBPS selection procedure ,IBPS Test time:2
hours,Reasoning,EnglishLanguage,Numerical Ability ,General Awareness with special
reference to Banking Industry ,Computer Knowledge solved question papers,IBPS
questions for practice.IBPS model question for practice,ibps free on line mock test,IBPS
clerical cadre examination notification and all details
IBPS clerical cadre Quantitative Aptitude Model Paper Questions Test 1 model
questions for practice
1.
By what number less than 1000 must 43259 be multiplied So that the last three
figures to the right of the product may be 437?
(a) 743 (b) 658 (c) 701 (d) None of these
Answer-a
2.
By what least number must 217800 be multiplied in order to make it a perfect square?
(a) 2 (b) 3 (c) 4 (d) None of these
Answer-a

3.
Find the least number by which 19404 must be multiplied or divided so as to make it a
perfect square.
(a) 9 (b) 10 (c) 11 (d) None of these
Answer-c
4.
Find the smallest number which is a perfect square and contains 1400 as a factor.
(a) 8670 (b) 9600 (c) 9000 (d) None of these
Answer-b
5.
Find in the blank indicated by a star in the number 456 so as to make it divisible by 33
(a) 3 (b) 4 (c) 5 (d) None of these
Answer-a
6.
By what least number must 6796 by multiplied or divided in order that the resulting
number may be a perfect square?
(a) 17 (b) 39 (c) 43 (d) None of these
Answer-b
7.
Find the greatest number that will divide 964, 1238 and 1400 leaving remainders
41,31 and 51 respectively.
(a) 71 (b) 64 (c) 67 (d) None of these
Answer-a
8.
Find the greatest number that will divide 19409 and 43037 leaving remainders 17 and
29 respectively.
(a) 178 (b) 192 (c) 194 (d) None of these
Answer-b
9.
Find the greatest number that will divide 964, 1238, 1400 leaving remainders 41, 31
and 51 respectively.
(a) 64 (b) 71 (c) 69 (d) None of these
Answer-b
10.
Find the greatest number which will divide 12288, 28200, and 44333 so as to leave
the same remainder in each case.
(a) 221 (b) 120 (c) 272 (d) None of these
Answer-a
Directions (questions 1 to 5): Read the following information and answer the
questions given below.
(i) A, B, C, D, E, F and G are seven members standing in a row (not necessarily in
the same order) facing north.
(ii) C and B have as many members between them as G and C have between them.
(iii) D, who is 3rd from the extreme left, is 3rd to the left of E.
(iv) A and D are neighbors and F and C are neighbors.

1.
Which of the following statements may be false?
(a) A is the 3rd to the left of F
(b) D is 3rd to the left of E
(c) F is 3rd the right of A
(d) B is 3rd to the left of C
(e) None of these
Ans (d)
2.
Which of the following statements is true?
(a) C and E are neighbors
(b) E is to the immediate left of F
(c) C is to the immediate left of D
(d) A is to the immediate left of D
(e) None of these
Ans (d)
3.
Who is at the extreme right?
(a) Data inadequate
(b) G
(c) B
(d) E
(e) None of these
Ans (a)
4.
Which of the following given two pairs of neighbors?
(a) A, C and D, C
(b) A, B and E, G
(c) D, C and E, F
(d) C, F and C, E
(e) None of these
Ans (c)
5.
What is the position of F?
(a) Immediate to the right of E.
(b) Third to the left of G
(c) Immediate to the left of C
(d) 5th to the right of B
(e) None of these
Ans (e)
6.
Four of the following five groups of letters are alike in some way while one is different.
Find out which one is different.
(a) ISLOJ
(b) LUOQM
(c) AKDGB

(d) FPILG
(e) NXQTO
Ans (b)
7.
In the sequence given below the sum of the two digits which immediately precede the
digit 4 exceeds the sum of the two digits which immediately follow the digit 4 and sum of
the two digits which immediately follow the digit 6 exceeds the sum of the two digits which
immediately precede the digit 6. How many such 4s and 6s together are there?
54462635642843766483
(a) 4
(b) 6
(c) 3
(d) 5
(e) None of these
Ans (a)
8.
Below are given six three-digit numbers. The digits comprise of numeric and letters.
The letter indicates its serial order in the English alphabet. What will be the middle digit of
the 4th number when the numbers are arranged in the descending order after interchanging
numeric in each number without altering the place of letter in the number?
19F, 2H9, 98B, D76, 7A6, 61E
(a) 8
(b) 9
(c) 6
(d) 7
(e) None of these
Ans (e)
Directions (Q.9 & 10): In a code language any letter which is immediately after or
before a vowel in the English alphabet is substituted by that vowel and any vowel
i.e. A, E, I, O and U is substituted by the letter immediately following that vowel in
the English alphabet.
9.
How can the word FEVERISH be written in that code language?
(a) EDVDRJSI
(b) EFUFRHSI
(c) EFUFRJSI
(d) EDUFRJSI
(e) None of these
Ans (c)
10.
How can the word CONFEDERATION be written in that code language?
(a) CPOGFEDRBUTPO
(b) CPOEFEFRBUJPO
(c) CNOEFEFRBUHNO
(d) CONFFDERATION
(e) None of these

Ans (b)
11.
If each of the vowels i.e. A, E, I, O, & U along with the 3rd letter to its right in the
alphabet are taken out and arranged one after the other in the same order followed by the
remaining letters of the alphabet, which of the following will be 5th to the left of the 19th
letter from the left in the new arrangement? A B C D E F G H I J K L M N O P Q R S T U V W
XYZ
(a) G
(b) H
(c) J
(d) W
(e) none of these
Ans (a)
12.
How many pairs of letters are there in the word SPONTANEOUS which have number
of letters between them in the word one less than the number of letters between them in
English alphabet?
(a) five
(b) one
(c) four
(d) Two
(e) Three
Ans (a)
13.
Four of the following five have similar relationship and hence form a group. Which
one does not belong to the group?
(a) BROTHER : DORVEHT
(b) ENGLISH : GGNNSIJ
(c) ANOTHER : CONVEHT
(d) BETWEEN : DTEZEEP
(e) HUSBAND : JSUDNAF
Ans (d)
14.
Which pair of the letters in the word BEAUTIFUL has the same relationship between
its letters with respect to their position in the English alphabet as the pair EA in that word
has between its letters?
(a) IB
(b) LF
(c) IE
(d) FL
(e) TL
Ans (c)
15.
Which of the following relates to FLOWER in the same way as RTERBN relates to
SECTOR?
(a) RWLGPF

(b) EOFKUQ
(c) EOFMXS
(d) RWLEPD
(e) RWLEND
Ans (e)
16.
If arranged properly which one of the following is not a river in India.
(a) ANUMAY
(b) IHSOK
(c) TABROC
(d) ADAMRAN
(e)None of these
Ans (c) It is CORBAT. It is a National Park
17.
If the following words are arranged in a natural sequence, which word would be the
first.
(a) Night
(b) Negative
(c) Neighbour
(d) Near
(e) None of these
Ans (d)
18.
In an office having 55 employees men out number the women by 15. How many
women are working in that office.
(a) 30
(b) 20
(c) 40
(d) 50
(e) None of these
Ans (b)
19.
Reaching a place of appointment on Friday I found that I was two days earlier than
the scheduled day. If I had reached on the following Wednesday how many days late would
I have been.
(a) one day
(b) two days
(c) three days
(d) four days
(e) None of these
Ans (c)
20.
Reena traveled from point A to a distance of 10 feet east at point B. She then turned
right and walked 3 feet. Again she turned right and walked 14 feet. How far is she from the
starting point.
(a) 4 feet

(b) 5 feet
(c) 24 feet
(d) 25 feet
(e) None of these
Ans (a)
Directions (Questions 21 to 25):- One of the words given in the alternatives
cannot be formed by using the letters given in the question. Find out that word.
21.
ENCOURAGE
(a) ANGER
(b) GREEN
(c) USAGE
(d) COURAGE
(d) None of these
Ans (c) There is no alphabet S in the question
22.
CONCENTRATE
(a) CENTRE
(b) CONCERN
(c) TREAT
(d) REASON
(e) None of these
Ans (d)
23.
INTRANSIGENT
(a) STAIN
(b) TRAIN
(c) RESIGN
(d) TRACE
(e) None of these
Ans (d)
24.
MAGNETIC
(a) MENACE
(b) GAIN
(c) NAME
(d) GAME
(e) None of these
Ans (a)
25. ENTHUSIASM
(a) MITE
(b) ASSIST
(c) ATHENS
(d) SENT

(e) None of these


Ans (b)
Directions (questions 26 to30 ):Read the following information and answer the
questions given below.
(i) A, B, C, D, E, F and G are seven members standing in a row (not necessarily in
the same order) facing north.
(ii) C and B have as many members between them as G and C have between them.
(iii) D, who is 3rd from the extreme left, is 3rd to the left of E.
(iv) A and D are neighbors and F and C are neighbors.
26.
Which of the following statements may be false?
(a) A is the 3rd to the left of F
(b) D is 3rd to the left of E
(c) F is 3rd the right of A
(d) B is 3rd to the left of C
(e) None of these
Ans (d)
27.
Which of the following statements is true?
(a) C and E are neighbors
(b) E is to the immediate left of F
(c) C is to the immediate left of D
(d) A is to the immediate left of D
(e) None of these
Ans (d)
28.
Who is at the extreme right?
(a) Data inadequate
(b) G
(c) B
(d) E
(e) None of these
Ans (a)
29.
Which of the following given two pairs of neighbors?
(a) A, C and D, C
(b) A, B and E, G
(c) D, C and E, F
(d) C, F and C, E
(e) None of these
Ans (c)
30.

What is the position of F?

(a) Immediate to the right of E.


(b) Third to the left of G
(c) Immediate to the left of C
(d) 5th to the right of B
(e) None of these
Ans (e)
31
Four of the following five groups of letters are alike in some way while one is different.
Find out which one is different.
(a) ISLOJ
(b) LUOQM
(c) AKDGB
(d) FPILG
(e) NXQTO
Ans (b)
32 . In the sequence given below the sum of the two digits which immediately precede
the digit 4 exceeds the sum of the two digits which immediately follow the digit 4 and
sum of the two digits which immediately follow the digit 6 exceeds the sum of the two digits
which immediately precede the digit 6. How many such 4s and 6s together are
there?
54462635642843766483
(a) 4
(b) 6
(c) 3
(d) 5
(e) None of these
Ans (a)
33.
Below are given six three-digit numbers. The digits comprise of numeric and letters.
The letter indicates its serial order in the English alphabet. What will be the middle digit of
the 4th number when the numbers are arranged in the descending order after interchanging
numeric in each number without altering the place of letter in the number?
19F, 2H9, 98B, D76, 7A6, 61E
(a) 8
(b) 9
(c) 6
(d) 7
(e) None of these
Ans (e)
Directions (Q.34 & 35): In a code language any letter which is immediately after
or before a vowel in the English alphabet is substituted by that vowel and any
vowel i.e. A, E, I, O and U is substituted by the letter immediately following that
vowel in the English alphabet.

34 . How can the word FEVERISH be written in that code language?


(a) EDVDRJSI
(b) EFUFRHSI
(c) EFUFRJSI
(d) EDUFRJSI
(e) None of these
Ans (c)
35 . How can the word CONFEDERATION be written in that code language?
(a) CPOGFEDRBUTPO
(b) CPOEFEFRBUJPO
(c) CNOEFEFRBUHNO
(d) CONFFDERATION
(e) None of these
Ans (b)
36 . If each of the vowels i.e. A, E, I, O, & U along with the 3rd letter to its right in the
alphabet are taken out and arranged one after the other in the same order followed by the
remaining letters of the alphabet, which of the following will be 5th to the left of the 19th
letter from the left in the new arrangement? A B C D E F G H I J K L M N O P Q R S T U V W
XYZ
(a) G
(b) H
(c) J
(d) W
(e) none of these
Ans (a)
37 . How many pairs of letters are there in the word SPONTANEOUS which have number
of letters between them in the word one less than the number of letters between them in
English alphabet?
(a) five
(b) one
(c) four
(d) Two
(e) Three
Ans (a)
38 . Four of the following five have similar relationship and hence form a group. Which
one does not belong to the group?
(a) BROTHER : DORVEHT
(b) ENGLISH : GGNNSIJ
(c) ANOTHER : CONVEHT
(d) BETWEEN : DTEZEEP
(e) HUSBAND : JSUDNAF
Ans (d)

39 . Which pair of the letters in the word BEAUTIFUL has the same relationship between
its letters with respect to their position in the English alphabet as the pair EA in that word
has between its letters?
(a) IB
(b) LF
(c) IE
(d) FL
(e) TL
Ans (c)
40 . Which of the following relates to FLOWER in the same way as RTERBN relates to
SECTOR?
(a) RWLGPF
(b) EOFKUQ
(c) EOFMXS
(d) RWLEPD
(e) RWLEND
Ans (e)
41 . If arranged properly which one of the following is not a river in India.
(a) ANUMAY
(b) IHSOK
(c) TABROC
(d) ADAMRAN
(e)None of these
Ans (c) It is CORBAT. It is a National Park
42 . If the following words are arranged in a natural sequence, which word would be the
first.
(a) Night
(b) Negative
(c) Neighbour
(d) Near
(e) None of these
Ans (d)
43.
In an office having 55 employees men out number the women by 15. How many
women are working in that office.
(a) 30
(b) 20
(c) 40
(d) 50
(e) None of these
Ans (b)

44.
Reaching a place of appointment on Friday I found that I was two days earlier than
the scheduled day. If I had reached on the following Wednesday how many days late would
I have been.
(a) one day
(b) two days
(c) three days
(d) four days
(e) None of these
Ans (c)
45 . Reena traveled from point A to a distance of 10 feet east at point B. She then turned
right and walked 3 feet. Again she turned right and walked 14 feet. How far is she from the
starting point.
(a) 4 feet
(b) 5 feet
(c) 24 feet
(d) 25 feet
(e) None of these
Ans (a)
Directions (Questions 46 to 25):- One of the words given in the alternatives
cannot be formed by using the letters given in the question. Find out that word.
46
ENCOURAGE
(a) ANGER
(b) GREEN
(c) USAGE
(d) COURAGE
(d) None of these
Ans (c) There is no alphabet S in the question
47.
CONCENTRATE
(a) CENTRE
(b) CONCERN
(c) TREAT
(d) REASON
(e) None of these
Ans (d)
48.
INTRANSIGENT
(a) STAIN
(b) TRAIN
(c) RESIGN
(d) TRACE
(e) None of these
Ans (d)

49.
MAGNETIC
(a) MENACE
(b) GAIN
(c) NAME
(d) GAME
(e) None of these
Ans (a)
50
ENTHUSIASM
(a) MITE
(b) ASSIST
(c) ATHENS
(d) SENT
(e) None of these
Ans (b)
Placement Paper 2737
IBPS PO CWE clerical and specialists officers previous years reasoning solved
question papers with answers and detailed explanations IBPS Aptitude,
Reasoning, General Awareness, Computer questions free solved sample placement
papers of all IT NON IT Bank PSU, all competitive examinations for learn and
practice, IBPS PO clerical,SBI and SBT current affairs and general awareness
multiple choice objective type questions with answers, What is the selection
procedure for bank recruitment, How to crack bank Interview,
IBPS Aptitude Reasoning Data Intrepretation questions with answers

1. If Atul finds that he is twelfth from the right in a queue of boys and fourth from the left,
how many boys should be added to the queue so that there are 28 boys in the queue ?
(a) 12
(b) 13

(ANS)

(c) 14
(d) 20
Hints & Solutions : From formula, total number of boys in the queue = (Atul's rank from
left) + (Atul's rank from right) - 1
= 12 + 4 - 1 = 15

Total number of boys that should be added

= 28 - 15 = 13.

2. Manish ranked sixteenth from the top and twenty ninth from the bottom among those
who passed an examination. Six boys did not participate in the competition and five failed
in it. How many boys were there in the class ?
(a) 40
(b) 44
(c) 50
(d) 55

(ANS)

Hints & Solutions : From formula, total number of passed students = (Manisha's rank
from the top) + (Manisha's rank from the bottom) - 1
= 16 + 29 - 1 = 4

Total number of boys in the class

= Passed students + failed students + absent students


= 44 + 6 + 5 = 55

3. Aruna ranks twelfth in a class of forty six. What will be her rank from the last ?
(a) 33
(b) 34
(c) 35

(ANS)

(d) 37
Hints & Solutions :

From formula,

Aruna's rank from the last = (total number of girls in the class) - (Aruna's rank from the
top) + 1
= 46 - 12 + 1 = 47 - 12 = 35th

4. Ravi is 7 ranks ahead of Sumit in a class of 39. If Sumit's rank is seventeenth from the
last, what is Ravi's rank from the start ?
(a) 14th
(b) 15th
(c) 16th

(ANS)

(d) 17th
Hints & Solutions : Ravi is 7 ranks ahead of Sumit. That means, 6 students are there
between Ravi and Sumit. Sumit's rank is 17th from the last.

Total number of students = 39


Total number of boys before Ravi = 39 - (16 + 1 + 6 + 1)
= 39 - 24 = 15

Ravi's rank is 16th from the start.

5. In a class of 60 where girls are twice that of boys, Kamal ranked seventeenth from the
top. If there are 9 girls a head of Kamal, how many boys are after him in rank ?
(a) 3
(b) 7
(c) 12

(ANS)

(d) 23

Directions (Qns. 6 to 11) : In each question below are given three statements followed
by four conclusions-I, II, III and IV. You have to take the given statements to be true even
if they seem to be at variance with commonly known facts. Read all the conclusions and
then decide which of the given conclusions and then decide which of the given conclusions
logically follow (s) from the given statements disregarding commonly known facts.
6. Statements :
Some books are pens.
All pens are chairs.
Some chairs are tables.
Conclusions :
I.

Some books are chairs.

II. All chairs are books.


III. All tables are chairs.
IV. Some tables are chairs
A) All follow
B) Only I, II and III follow
C) Only I, II and IV follow
D) Only II, III and IV follow
E) None of these

(Ans)

Explanatory Ans : - First and third premises are Particular Affirmative, i.e. I-type.
Second premise is Universal Affirmative (A-type).

We know that,
I + A 1 - type conclusion Therefore, our derived conclusion would be :
"Some books are chairs" This is the conclusion I.
Conclusion II is the conversion of our derived conclusion.
Two out of three premises are Particular and hence, Universal conclusion is invalid. This is
conclusion III does not follow. Conclusion IV is the conversion of the third premise.
Therefore, only conclusions I, II and IV follow.

7. Statements :
All cars are jeeps.
All Jeeps are buses.
All buses are trucks.
Conclusions :
I.

All trucks are buses.

II.

All buses are jeeps.

III. All jeeps are cars.


IV. All cars are trucks
A) None follows
B) All follow
C) Only III and IV follow
D) Only IV follows

(Ans)

E) None of these
Explanatory Ans : -

All the three premises are Universal Affirmative (A-type).

We know that,
A + A A-type conclusion
Therefore, our required conclusion would be:
"All cars are buses".

There is no such conclusion.


Now,

All buses are trucks.


A + A A - type conclusion
Thus, our derived conclusion would be :
"All cars are trucks".
This is the conclusion IV.
Again
We know that,
A + A A - type conclusion
Thus, our derived conclusion would be :
"All jeeps are trucks" Therefore, only conclusion IV follows. Thus, our required answer is
option (D).
VENN - DIAGRAM

8 Statements :
Some trees are flowers.
Some flowers are pencils.
Some pencils are tables.
Conclusions :
I.

Some tables are flowers.

II.

Some pencils are trees.

III. Some tables are trees.


IV. Some trees are pencils.
A) All follow
B) None follows

(Ans)

C) Only I and III follow


D) Only II and IV follow
E) None of these

Explanatory Ans : - All the three premises are Particular Affirmative (I-type). Therefore,
no conclusion can be derived from these premises. Now look for any conversion and/or
implication :
There is no such conclusion.

9.

Statements :

All rods are bricks


Some bricks are ropes
All ropes are doors
Conclusions :
I.

Some rods are doors.

II.

Some doors are bricks.

III. Some rods are not doors.


IV. All doors are ropes.
A) Only I and II follow
B) Only I, II and III follow
C) Only either I or III and II follow

(Ans)

D) Only either I or III and IV follow


E) None of these
Explanatory Ans : - First premise is Universal Affirmative (A-type).
Second premise is Particular Affirmative (I-type).
Third premise is Universal Affirmative (A-type).
We know that,
I + A I - type conclusion
Conclusions : Some bricks are doors.
It is conversion of conclusion II. Conclusions I and III form complementary pair. Therefore
either conclusion I or III follows. Therefore, our required answer is option (C).

10. Statements :
Some books are pens.
Some pens are watches.

All watches are radios.


Conclusions :
I.

Some radios are watches.

II.

Some radios are pens

III. Some watches are books


IV. Some books are watches
A) All follow
B) Only I and III follow
C) Only II and IV follow
D) Only I and IV follow
E) None of these

(Ans)

Explanatory Ans : - First and second premises are Particular Affirmative, i.e. I-type. Third
premise is Universal Affirmative (A-type). Conclusion I is the conversion of the third
premise.
Second and third premises are relevant for the conclusion II.
Thus,

We know that,
I + A I - type conclusion
Conclusions : Some pens are radios.
Conclusion II is the conversion of this conclusion.
Therefore, only conclusions I and II follow.

11. Statements :
All towns are villages.
No village is forest.
Some forests are rivers.
Conclusions :
I.

Some forests are villages.

II.

Some forests are not villages.

III. Some rivers are not villages.

IV. All villages are towns.


A) All follow
B) Only either I or II follows
C) Only either I or II and III follow
D) None follows
E) None of these

(Ans)

Explanatory Ans : - First premise Universal Affirmative (A-type).


Second premise Universal Negative (E-type).
Third premise Particular Affirmative (I-type).
Now,

We know that,
A + E E - type conclusion
Conclusions :
No town is forest.
There is no such conclusion.
We know that,
E + I O1 - type conclusion
Thus, our derived conclusion would be :
"Some rivers are not villages"
This is the conclusion III.
Again,

We now that,
E + I O1 - type conclusion
Conclusions :
Some rivers are not towns,
There is no such conclusion.
Conclusion II is the conversion of the second premise.
Therefore, Conclusions II and III follow.

12 In a row of boys facing north, Sudhanshu is twelfth from his left. When shifted to his
right by four places, he becomes eighteenth from the right end of the row. How many boys
are there in the row ?
A) 32
B) 33

(Ans)

C) 34
D) Data inadequate
E) None of these None of these

After shifting his place


Total number of boys in the row = (16 + 18) - 1 = 33

13. In a certain code language PROBLEM is written as MPERLOB. How will NUMBERS be
written in that code ?
A) SNUREMB
B) SNRUBME
C) SNRUEMB

(Ans)

D) SNRUMEB
E) None of these
Explanatory Ans : 1
P

2
R

3
O

4
B

5
L

6
E

7
M

It has been coded as


7
M

1
P

6
E

2
R

5
L

3
O

4
B

4
B

5
E

6
R

7
S

2
E

5
M

3
B

4
S

Similarly,
1
N

2
U

3
M

Its code would be :


7
N

1
R

6
U

Directions (Qns. 14 to 40) : In each question below is given a statement followed by two
assumptions numbered I and II. An assumption is something supposed or taken for
granted. You have to consider the statement and the following assumptions and decide
which of the assumptions is implicit in the statement.
Give answer (A) if only assumption I is implicit.
Give answer (B) if only assumption II is implicit.
Give answer (C) if either I or II is implicit.
Give answer (D) if neither I nor II is implicit.
Give answer (E) if both I and II are implicit.

14. Statement : The government has recently hiked the prices of diesel and petrol to
reduce the oil poor deficit.
Assumptions :
I.

The amount earned by this increase may be substantial enough to reduce the deficit.

II. There may be wide spread protests against the price hike.
Explanatory Ans : -

(Ans : E)

Both the assumption are implicit in the statement.

It is clearly mentioned in the statement that the Government has hiked the prices of diesel
and petrol to reduce the oil pool deficit.
Whenever the prices of commodities are hiked, generally people raised the voice against
such measure.

15. Statement : The X passenger car manufacturing company announced a sharp


reduction in the prices of their luxury cars.
Assumptions :
I.
II.

There may be an increase in the sale of their luxury cars.


The other such care manufacturers may also reduce their prices.

Explanatory Ans : (Ans : E) The price of any product is lowered assuming that its
lowered assuming that its demand will increase. Therefore assumption I is implicit in the
statement.

16. Statement : A foreign film producer rendered his apology before Indian society for
misinterpreting a part of Indian epic.
Assumptions :
I.

Indians are very sensitive to the misinterpretation of their epic.

II.

It is possible to derive wrong meaning from the epic.

Explanatory Ans : (Ans : E) From the content of the statement it is clear that both
the assumptions re implicit in the statement.

17. Statement : Aswin's mother instructed him to return home by train if it rains heavily.
Assumptions :
I.
II.

Aswin may not be able to decide himself if it rains heavily.


The trains may ply event if it rains heavily.

Explanatory Ans : (Ans : B) Only assumption II is implicit in the statement. If


Aswin's mother asked his son to return home by train if it rains heavily, it implies that the
trains would ply even if it rains heavily.

18. Statement : The Government of India has decided to start a track II dialogue with its
neighbor to reduce tension in the area.
Assumptions :
I.

The neighboring country may agree to participate in the track II dialogue.

II. The people involved in track II dialogue may be able to persuade their respective
Governments.
Explanatory Ans : -

(Ans : E)

Both the assumptions are implicit in the statement.

19 Statement : The host in one of the popular T.V programmer announced that the
channel will contact the viewers between 9.00 a.m. to 6.00 p.m. on weekdays and the
lucky ones will be given fabulous prizes.
Assumptions :
I.
II.

The people may remain indoors to receive the phone call.


More people may start watching the Programme.

Explanatory Ans : -

(Ans : E)

Both the assumptions are implicit in the statement

IBPS Bank PO (CWE) Reasoning Questions with answers 9818


Question papers of IBPS Bank PO and Clerical cadre Written Test Examination Pattern and
Latest Question Papers\
IBPS stands for Institute of Banking Personnel Selection Common Written Examination
(CWE) for PO,Clerical and Gramin Banks Officer scale jobs
IBPS PO (CWE) Gramin Banks Latest Selection Procedures2012|Aptitude Reasoning, Data
Interpretation computer Knowledge and General Awareness Questions
IBPS Previous Years question papers with answers Detailed Test Analysis|IBPS

Hi
These are the some model reasoning questions with answers previoulsy asked in different
bank tests.
IBPS Bank PO CWE Reasoning Questions
IBPS Bank PO CWE Coding and decoding,blood relation,datasufficiency questions........

1. BE' is related to 'GJ' in the same way as 'PS' is related to


1) UY
2) UX
3) UZ
4) VY
5) None of these Ans. 1 (2)

2. Among P, Q, R, S and T, each having scored different marks, R scored more marks than P
and T. Q scored less marks than T. S did not score the highest marks. Who among them
scored the highest?
1) P
2) T
3) R
4) Data inadequate
5) None of these Ans. 2 (3)

3. Pointing to a girl, Nidhi said, 'She is the daughter of my grandmother's only child.' How is
the girl related to Nidhi?
1) Sister
2) Self
3) Cousin sister
4) Data inadequate
5) None of these Ans.3 (4)

4. In a certain code MOAN is written as 5%3$ and NEWS is written as $1@8. How is SOME
written in that code?
1) 8%o51
2) 85%8
3) 8@51

4) 8%31
5) None of these Ans.4 (1)

5. In a certain code language 'how many books' is written as 'sa da na' and 'many more
days' is written as 'ka pa da'. How is 'books' written in that code language?
1) sa
2) na
3) sa or na
4) Data inadequate
5) None of these Ans 5)(3)
6. In a certain code MAJORITY is written as 'PKBNXSHQ'. How is SANCTION written in that
code?
1) TBODMNHS
2) DOBTMNHS
3) TBODSHNM
4) DOBTOPJU
5) None of these Ans.6 (2)
7. The position of the first and the sixth digits in the number 5120397468 are interchanged,
similarly the positions of the second and the seventh digits are interchanged and so on.
Which of the following will be the fourth digit from the right end after the rearrangement?
1) 1
2) 5
3) 7
4) 9
5) None of these Ans.7 (1)
8. How many such pairs of letters are there in the word OROANISED each of which has as
many letters between them in the word as in the English alphabet?
1) None
2) One
3) Two
4) Three
5) More than three Ans. 8 (3)

9. How many meaningful English words can be made with the letters ITRM using each letter
only once in each word?
1) None
2) One
3) Two
4) Three
5) More than three Ans. 9 (2)

10. The position of how many digits in the number 8247531 will remain unchanged after the
digits are rearranged in ascending order within the number?
1) None
2) One
3) Two
4) Three
5) More than three Ans.10 (3)

11. In a row of twenty-five children facing North, W is fifth to the right of R, who is
sixteenth from the right end of the row. What is W's position from the right end of the row?
1) Eleventh
2) Tenth
3) Twelfth
4) Data inadequate
5) None of these Ans 1)
Directions (Q. 12-14): Study the following, information carefully and answer the questions,
which follow:
Five plays A, B, C, D and E were organized in a week from Monday to Saturday with one
play each day and no play was organised on one of these days. Play D was organised before
Thursday but after Monday. Play E was organised on Saturday. Play C was not organised on
the first day. Play B was organized on the next day on which play C was organised. Play A
was organised on Tuesday.
12. On which day was play B organised?
1) Thursday
2) Friday
3) Wednesday
4) Data inadequate
5) None of these Ans.12 (2)
13. On which day was no play organised?
1) Monday
2) Wednesday
3) Thursday
4) Data inadequate
5) None of these
Ans.13 (1)

14. Which play was organised on Wednesday?


1) A
2) C
3) D

4) Data inadequate
5) None of these
Ans.14 (3)
15. If means '+'; 'x' means '-',; '+' means 'x and '-' means '', then 24 -46 x3+4 =?
1) 36
2) 24
3) 8
4) 4
5) None of these
Ans.15 (5)
16. What should come next in the following number series?
98761234598712345987123498
1) 7
2) 1
3) 2
4) 5
5) None of these
Ans.16 (2)
17. Each consonant in the word CORDIAL is changed to the previous letter in the English
alphabet and each vowel is changed to the next letter in the English alphabet and the letters
so obtained are rearranged in alphabetical order. Which of the following will be the second
from the right end after the rearrangement?
1) Q
2) K
3) P
4) B
5) None of these
Ans.17 (3)
18. Meena correctly remembers that her father's birthday is after eighteenth May but before
twentysecond May. Her brother correctly remembers that their father's
Birthday is before twenty-fourth May but after Twentieth May. On which date in May was
definitely their father's birthday?
1) Twentieth
2) Nineteenth
3) Eighteenth
4) Cannot be determined
5) None of these
Ans.18 (5)
Directions (Q. 19-20): Study the following information and answer the questions given
below:

(i) 'P Q' means 'P is sister of Q'.


(ii) 'P x Q' means 'P is brother of Q'.
(iii) 'P - Q' means 'P is mother of Q'.
(iv) 'P + Q' means 'P is father of Q'.
19. Which of the following means 'M is maternal uncle of T?
1) MK+T
2) M x K+T
3) M x K-T
4) M K - T
5) None of these
Ans.19 (4)
20. Which of the following means 'H is paternal grandfather of T?
1) H +J+T
2) T x K + H
3) H +J x T
4) H - J + T
5) None of these
Ans.20 (1)
Directions (Q. 21-25): Following questions are based on the five three-digit numbers given
below:
832 965 748 259 614
21. Which of the following is the third digit of the second lowest number?
1) 2
2) 5
3) 8
4) 9
5) 4
Ans. 21 (5)
22. Which .of the following is the sum of the first' and the second digits of the highest
number?
1) 11
2) 15
3) 10
4) 7
5) None of these
Ans.22 (2)
23. If the positions of the second and the third digits of each of the numbers are
interchanged, which of the following will be the last digit of the lowest number?
1) 5
2) 1
3) 6
4) 4

5) 3
Ans.23 (1)
24. If the positions of the first and the second digits of each of the numbers are
interchanged, which of the following will be the second digit of the second highest number?
1) 8
2) 6
3) 7
4) 5
5) None of these
Ans.24 (5)
25. Which of the following is the difference between the first and the third digits of the third
highest number?
1) 6
2) 4
3) 1
4) 7
5) None of these
Ans.25 (3)

Directions (Q. 26-31): In each of the questions below is given four statements followed by
three conclusions numbered I, II and m. You have to take the given statements to be true
even if they seem to be at variance with commonly known facts. Read all the conclusions
and then decide which of the given conclusions logically follows from the given statements
disregarding commonly known facts.
26. Statements:All stones are poles.
All poles are desks.
Some desks are nets.
All nets are days.
Conclusions: I. Some nets are stones.
II.Some desks are stones.
III.Some days are desks.
1)
2)
3)
4)
5)

Only I and II follow


Only I and III follow
Only II and III follow
All I, II and III follow
None of these

Ans.26 (3)

27. Statements: Some months are weeks.


Some weeks are years.
All years are buses.
All buses are trains.
Conclusions: I. Some trains are weeks.
II. Some buses are weeks.
III.Some trains are months.
1) Only I and II follow
2) Only II and III follow
3) Only I and III follow
4) All, I, II and III follow
5) None of these
Ans.27 (1)
28. Statements: Some stations are rails.
All rails are rivers.
All rivers are papers.
Some papers are cards.
Conclusions: I. Some cards are stations.
II.Some rivers are stations.
III. Some cards are rivers.
1) Norte follows
2) Only I follows
3) Only II follows
4) Only III follows
5) Only II and III follow
Ans.28 (3)
29. Statements: All windows are roofs.
All roofs are glasses.
Some glasses are plates.
Some plates are carpets.
Conclusions: I. Some plates are windows.
II. Some glasses are windows.
III. Some carpets are roofs.
1) None follows
2) Only I follows
3) Only II follows
4) Only III follows
5) Only I and II follow
Ans.29 (3)
30. Statements: All trees are jungles.

All jungles are houses.


All houses are buildings.
All buildings are villages.
Conclusions: I. Some villages are houses.
II. Some buildings are jungles.
III. Some houses are trees.
1) Only I and II follow
2) Only II and III follow
3) Only I and III follow
4) All I, II and III follow
5) None of these
Ans.30 (4)
31. Statements: Some books are calendars.
No calendar is a paper.
All papers are pencils.
Some pencils are chairs.
Conclusions: I. Some chairs are books.
II. No chair is a book.
III. Some pencils are calendars.
1) None follows
2) Only either I or II follows
3) Only either I or II and III follow
4) Only III follows
5) Only n follows
Ans.31 (2)
Directions (Q. 32-37): Study the following arrangement and answer the questions given
below:
M5%PU2A$43ZEK 19QRI@D7F88WN6#VJ*Y
32. Which of the following is the ninth to the right of the seventeenth from the right end of
the above arrangement?
1) N
2) $
3)4
4) W
5) None of these
Ans.32 (1)
33. Which of the following is the fourth to the left of the sixth to the left of W in the above
arrangement?
1) U
2) D
3) E

4)
5) None of these Ans.33 (5)

34. How many such symbols are there in the above arrangement each of which is
immediately preceded by a letter and immediately followed by a number?
1) None
2) One
3) Two
4) Three
5) More than three
Ans.34 (3)
35. How many such consonants are there in the above arrangement each of which is
immediately followed by a symbol but not immediately preceded by a number?
1) None
2) One
3) Two
4) Three
5) More than three
Ans.35 (3)
36. Four of the following five are alike in a certain way based on their positions in the above
arrangement and so form a group. Which is the one that does not belong to that group?
1) P5A
2) 4AE
3) QIK
4) F8@
5) VN*
Ans.36 (5)
37. If all the symbols and the numbers in the above arrangement are dropped, which of the
following will be the twelfth from the left end?
1) E
2) K
3) Z
4) D
5) None of these
Ans.37 (5)
Directions (Q. 38-43): In each question below is given a group of digits/symbols followed by
four combinations of letters numbered (1), (2), (3) and (4). You have to find out which of
the combinations correctly represents the group of digits/symbols based on the following
coding system and the conditions that follow and mark the number of that combination as
your answer. If none of the combinations correctly represents the group of digits/symbols,

mark (5), ie 'None of these' , as your answer. Digit/Symbol : 5 @ 3 9 8 % 8 1 4 2 $ # 7 6


* Letter Code : P I MAE G F J H B D. N R Q T U .
Conditions: (i) If the first unit is a symbol and the last unit is an even digit both are to be
coded as the code for the even digit.
(ii) If the first unit is all even digit and the last unit is an odd digit their codes are to be
interchanged.
(iii) If the first unit is an odd digit and the last unit is a symbol both at e 1:0 be coded as
the code for the symbol.
(iv) If both the first and the last units are symbols both are to be coded as Z.
38. 9@374%
1) EIMQBF
2) FBQMIE
3) ZIMQBZ
4) FIMQBF
5) None of these
Ans.38 (4)

39. 29$#1
1) HEANRD
2) DEANRH
3) DEANRD
4) HEANRH
5) None of these Ans.39 (1)
40. *479@
1) UBQEIA
2) ZBQEIA
3) ZBQEIZ
4) ABQEIU
5) None of these Ans.40 (3)
41. % 286$3
1) FDGTNF
2) FDGTNM
3) FGDTNM
4) WGTNZ
5) None of these Ans.41 (2)
42. 54#*@6
1) PBURIT
2) PBRUIP
3) TBRUIP

4) TBRUIT
5) None of these Ans.42 (5)

43. $6394
1) NTMAEN
2) BTMAEB
3) ZTMAEB
4) NTMAEB
5) None of these Ans.43 (2)
Directions (Q. 44-48): Study the following information carefully and answer the questions
given below:
A, B, C, D, E, F, G and H are sitting around a circle facing the centre. F is second to the right
of A and third to the left of C. B is second to the left of C and fourth to the right of H. D is
second to the right of G
44. Who is to the immediate left of D?
1) H
2) C
3) G
4) Data inadequate
5) None of these Ans.44 (2)
45. Who is second to the right of E?
1) B
2) G
3) H
4) Data inadequate
5) None of these Ans.45 (1)
46. Who is third to the left of B?
1) E
2) H
3) F
4) Data inadequate
5) None of these Ans.46 (5)
47. What is the position of G with respect to A?
1) Third to the left
2) Third to the right
3) Fifth to the left

4) Fourth to the right


5) Fifth to the right Ans.47 (4)
48. In which of the following combinations is the third person sitting in between the first and
the second persons?
1) BGC
2) EFB
3) DAH
4) AEF
5) GCD Ans.48 (3)
Directions (Q. 49-54): In the following questions, the symbols @, , $, % and # are used
with the following meaning as illustrated below:
'P $ Q' means 'P is not greater than Q'.
'P@Q' means 'P is neither smaller than nor equal to Q'.
'P # Q' means' P is not smaller than Q'.
'P Q' means 'P is neither greater than nor equal to Q'.
'P % Q' means 'P is neither smaller than nor greeter than Q'.
Now in each of the following questions assuming the given statements to be true, find which
of the three conclusions I, II and III given below them is/are definitely true and give your
answer accordingly.
49. Statements: D # K K @ T, T $ M,. M '% .T
Conclusions: I. J @ T
II. J%T
III. D@T
1) Only I is true
2) Only II is true
3) Only either I or II is true
4) Only IV is true
5) Only either I or II and III are true Ans.49 (5)
50. Statements: R@N,ND, O$J, J#B
Conclusions: I. R @ J
II. J@N
III. B@D
1) None is true
2) Only I is true
3) Only II is true
4) Only III is true
5) Only I and III are true Ans.50 (3)

51. Statements: W B, B % V, V $ R, R@K


Conclusions: I. K B
II. R#B
III.V@W
1) Only I and II are true
2) Only I and III are true
3) Only II and III are true
4) AII I, II and III are true
5) None of these Ans.51 (3)

52. Statements: H $ M, M # T, T @ D, D R
Conclusions: I. D M
II. R@M
III. H$T
1) None is true
2) Only I is true
3) Only II is true
4) Only III is true
5) Only I and II are true Ans.52 (2)

53. Statements: B % J, J @ K, K T, T $ F
Conclusions: I. F @ K
II. B@K
III.B@F
1) Only I and II are true
2) Only I and III are true
3) Only II and III are true
4) All I, II and III are true
5) None of these Ans.53 (1)

54. Statements: F # B, B $ M, M @ K, J< e N


Conclusions: I. N @ M
II. F$M
III. KB
1) Only I is true
2) Only II is true
3) Only III is true
4) Only II and III are true
5) None is true Ans.54 (5)

Directions (55-60): Study the following information carefully and answer the given
questions:
A word and number arrangement machine when given an. input line of words and numbers
rearranges them following a particular rule in each step. The following is an illustration of
input and rearrangement.
Input : sale data 1823 for 95 then 38
Step I : data sale 1823 for 95 then 38
Step II : data 95 sale 18 23 for then 38
Step III : data 95 for sale 18 23 then 38
Step IV : data 95 for 38 sale 18 23 then
Step V : data 95 for 38 sale 23 18 then
Step VI : data 95 for 38 sale 23 then 18
and step VI is the last step of the rearrangement of the above input.
As per the rules followed in the above steps, find out in each of the following questions the
appropriate step for the given input.
55. Input: year 39 stake47 house full 94 55 How many steps will be required to complete
the rearrangement?
1) Five
2) Six
3) Four
4) Seven
5) None of these Ans.55 (2)
56. Step II of an input is: car 83 lost ever 324674 now How many more steps will be
required to complete the rearrangement?
1) Three
2) Four
3) Five
4) Six
5) None of these Ans.56 (2)
57. Step III of an input is : and 79 code 27 18 new goal 34 Which of the following will
definitely be the input?
1) code and 79 27 18 new goal 34
2) code 27 18 new goal 34 and 79
3) code 27 and 1879 new goal 34
4) Cannot be determined
5) None of these Ans.57 (4)

58. Input: water full never 35 78 16 height 28 Which of the following steps will be the last?
1) VI
2) VII
3) VIII
4) IX
5) None of these
Ans.58 (1)

59. Step II of an input is: high 69 store pay 36 43 15 there Which of the following will be
step VI?
1) high 69 pay 43 store 36 there 15
2) high 69 pay 43 store 36 15 there
3) high 69 pay 36 43 store 15 there
4) There will be no such step
5) None of these Ans.59 (4)
60. Input: train more 2953 fast gear 3784 Which of the following steps will be the last but
one?
1) IX
2) VIII
3) VII
4) VI
5) None of these
Ans.60 (5)

Directions (Q. 61-70): Study the following information carefully and answer the questions
given below:
Following are the conditions for selecting Assistant General Manager-HR in an organisation.
The candidate must
(i) be at least 40 years and not more than 50 years old as on 01-05-2010.
(ii) be postgraduate in Personnel
Management /HR with at least 60 per cent marks.
(iii) have post-qualification work experience of at least fifteen years out of which at least
five years as Senior Manager-HR in an organization.
(iv) have successfully implemented HR-System in his/ her organisation sometime during the
past three years.
(v) have secured at least 45 per cent marks in the selection process.
In the case of a candidate who satisfies all the conditions EXCEPT at
(A) (ii) above but has secured at least 50 per cent marks in postgraduation and at least 65
per cent marks in the selection process, the case is to be referred to Executive Director.
(B) (iii) above but has at least twelve years' post qualification work experience out of which
at least eight years as Senior Manager-HR in an organisation, the case is to be referred to

Chairman.
In each question below details of one candidate are given. You have to take one of the
courses of action based on the information provided and the conditions and sub-conditions
given above and mark the number of that course of action as your answer. You are not to
assume anything other than the information provided in each question. All these cases are
given to you as on 01-0572010.
Mark answer 1) if the candidate is to be selected.
Mark answer 2) if the candidate is not to be selected.
Mark answer 3) if the data provided are not adequate to take a decision.
Mark answer 4) if the case is to be referred to Executive Director.
Mark answer 5) if the case is to be referred to Chairman.

61. Pranab Ghosh was born on 8th March 1968. He has been working for the past eighteen
years in an organisation out of which last seven years as Senior Manager-HR after
completing his postgraduation in HR with 68 per cent marks. He had successfully
implemented HR-System last year in his organisation. He has secured 50 per cent marks in
the selection process.
62. Sheetal Jha has been working in an organisation for the past twenty years out of which
ten years as Senior Manager-HR after completing her postgraduation in Personnel
Management with 70 per cent marks. She was born on 2nd December 1965. She has
secured 45 percent marks in the selection process.
63. Prabir Sengupta was born on 8th May 1963. He has secured 65 percent marks in the
selection process. He has been working for the past fifteen years in an organisation, out of
which twelve years as Senior Manager HR, after completing his post graduation in HR with
55 per cent marks. He has successfully implemented HR-System in his organisation during
the last two years.
64. Shailesh Kumar has been working in an organisation for the past thirteen years, out of
which nine years as Senior Manager-HR after completing his postgraduation in HR with 68
per cent marks. He was born on 15th September 1968. He has secured 48 per cent marks
in the selection process. He has successfully implemented HR-System in his organization
two years back.
65. Navin Chopra was born on 12th June 1967. He has been working for the past sixteen
years, out of which seven years as Senior Manager-HR after completing his post-graduation
in Personnel Management with 75 per cent marks. He has secured 44 per cent marks in the
selection process. He has successfully implemented HR-System in his organisation last year.
66. Meera Kulkarni has been working for the past seventeen years, out of which eight years
as Senior Manager-HR, after completing her postgraduation in Personnel Management with
66 per cent marks. She has successfully implemented HR-System in her organisation during
the last two years. She has secured 49 per cent marks in the selection process. She was
born on 14th December 1971.

67. Akash Shastri was born on 12th April 1967. He has been. working for the past sixteen
years, out of which six years as Senior Manager-HR, in an organisation after completing his
postgraduation in HR with 58 per cent marks. He has successfully implemented HR-System
in his organization last year. He has secured 65 per cent marks in the selection process.
68. Shekhar Jena has been working for the past fifteen years, out of which last seven years
as Senior Manager-HR, in an organisation after completing his postgraduation in HR with 68
per cent marks. He has secured 60 per cent marks in the selection process. He was born on
16th.
69. Sunetra Govil was born on 5th April 1964. She has been working for the past seventeen
years, out of which nine years as Senior Manager-HR, in an organisation. She has secured
48 per cent marks in the selection process. She has also secured 69 per cent marks in her
post- graduation in Personnel Management. She successfully implemented HR-System in her
organisation last year.
70. Mohit Saxena was born on 27th July 1963. He has been working for the past thirteen
years, out of which nine years as Senior Manager-HR, after completing his post- graduation
in HR with 67 per cent marks. He has secured 49 per cent marks in the Selection process.
He has successfully implemented HR-System in his organisation during the past three years.
Answers :(Q.61-70)
Q.No. Person (i) (ii)/(A) (iii)/(B) (iv) (V) Ans
61. Pranab 1
62. Sheetal ? 3
63. Prabir () 4
64. Shailesh () 5
65. Navin 2
66. Meera 2
67. Akash () 4
68. Shekhar 1
69. Sunetra ? 3
70. Mohit () 5
Directions (Q. 71-75): Below in each question are given two statements (A) and (B). These
statements may be either independent causes or may be effects of independent causes or a
common cause. One of these statements may be the effect of the other statement. Read
both the statements and decide which of the following answer choices correctly depicts the
relationship between these two statements.
Mark answer 1) if statement (A) is the cause and statement (B) is its effect.
Mark answer 2) if statement (B) is the cause and statement (A) is its effect.
Mark answer 3) if both the statements (A) and (B) are independent causes.
Mark answer 4) if both the statements (A) and (B) are effects of independent causes.
Mark answer 5) if both the statements (A) and (B) are effects of some common cause.
71. (A) The Govt has marginally increased the procurement price of wheat for the current
crop.
(B) The current wheat crop is expected to be twenty per cent more than the previous wheat
crop.

Ans.71 (1)
72. (A) The municipal authority demolished the tea stall located on the footpath on the busy
road.
(B) A large number of people have been taking their evening tea at the tea stall located on
the footpath on the main road, blocking pedestrian movement.
Ans.72 (2)
73. (A) Majority of the students left the local school as the school building was in a
dilapidated condition.
(B) The school authority decided to close down the school immediately and shift the
remaining students to a make-shift school.
Ans.73 (1)
74. (A) The braking system of the tourist bus carrying 40 passengers failed while
negotiating a stiff climb on a hilly road.
(B) The tourist bus fell into the gorge killing at least ten passengers and seriously injuring
all the remaining.
Ans.74 (1)
75. (A) The state govt has decided to boost English language education in all the schools
from the next academic year.
(B) The level of English language of the school students of the State is comparatively lower
than that of the neighbouring states.
Ans.75 (2)

IBPS Bank PO (CWE) Reasoning Question Paper 7631


IBPS Bank PO CWE Reasoning Questions
REASONING

IBPS Bank PO CWE Coding and Decoding Questions

1. In a certain code language how many goals scored is written as 5 3 9 7 : many more
matches is written as 9 8 2 and he scored five is written as 1 6 3. How is goals written
in that code language ?
A. 5 B. 7 C.5 or 7 D. Data Inadequate E. None of these

2. In a certain code TEMPORAL is written as OLDSMBSP. How is CONDENSER written in that


code ?
A. RMNBSFEJ B. BNMRSFEJ C.RMNBJEFS D. TOPDQDCH C. RMNBJEFS D.TOPDQDCH E.
None of these

3. How many meaningful English words can be made with the letters DLEI using each letter
only once in each word ?
A. None B. One C. Two D. Three E. More than three

4. Among A, B, C, D and E each having different weight, D is heavier than only A and C is
lighter than B and E. Who among them is the heaviest ?
A. B B. E C. C D. Data Inadequate E. None of these

5. Each odd digit in the number 6263187 is substituted by the next higher digit and each
even digit is substituted by the previous lower digit and the digits so obtained are
rearranged in ascending order, which of the following will be the third digit from the left and
after the rearrangement ?
A. 2 B. 4 C. 5 D. 6 E. None of these

6. Pratap correctly remembers that his mothers birthday is before twenty third April but
after Nineteenth April, whereas his sister correctly remembers that their mothers birthday
is not on or after twenty second April. On which day in April is definitely their mothers
birthday ?
A. Twentieth B. Twenty first C. Twentieth or Twenty first D. Cannot be determined E.
None of
these

7. Ashok started walking towards South. After walking 50 meters he took a right turn and
walked 30 meters. He then took a right turn and walked 100 meters. He again took a right
turn and walked 30 meters and stopped. How far and in which direction was he from the
starting point ?
A. 50 meters South B. 150 meters North C. 180 meters East D. 50 meters North E. None of
these

8. If means + ; - means x ; x means and + means -; then ________ 15-8 x 6


12+ 4 = ?
A. 20 B. 28 C. 8 4/7 D.2 2/3 E. None of these

9. Town D is toward East of town F. Town B is towards North of town D. Town H is towards
South of town B. Towards which direction is town H from town F ?
A. East B. South-East C. North-East D. Data inadequate E. None of these

10. How many such pairs of letters are there in the word SEARCHES each of which has as
many letters between them in the word as in the English alphabet ?
A. None B. One C. Two D. Three E. More than three

IBPS Bank PO CWE Logical Deductions Questions


Directions (Qs. 11-15) : In each of the questions below are given four statements followed
by four conclusions numbered I, II, III and IV. Your have to take the given statements to be
true even if they seem to be at variance from commonly known facts. Read all the
conclusions and then decide which of the given conclusions logically follows from the given
statements disregarding commonly known facts.
11.Statements :
All cups are bottles
Some bottles are jugs
No jug is plate
Some Plates are tables
Conclusions :
I. Some tables are bottles.
II. Some plates are cups.
III. No table is bottle.
IV. Some jugs are cups.
A. Only I follows
B. Only II follows
C. Only III follows
D. Only IV follows
E. Only either I or III follows

12. Statements :
Some Chairs are handles.
All Handles are pots
All pots are mats.
Some mats are buses.
Conclusions :
I. Some buses are handles.
II. Some mats are chairs.

III. No bus is Handle.


IV. Some mats are handles
A. Only I, II and IV follow
B. Only II, III and IV follow
C. Only either I or III and II follow
D. Only either I or III and IV follow
E. Only either I or III and II and IV follow

13. Statements:
All birds are horses.
All horses are tigers.
Some tigers are lions.
Some lions are monkeys.
Conclusions :
I. Some tigers are horses.
II. Some monkeys are birds.
III. Some tigers are birds.
IV. Some monkeys are horses.
A. Only I and III follow
B. Only I, II and IV follow
C. Only II, III and IV follow
D. All I, II, III and IV follow
E. None of these

14. Statements :
Some benches are walls.
All walls are houses.
Some houses are jungles.
All jungles are roads.
Conclusions :
I. Some roads are benches.
II. Some jungles are walls.
III. Some houses are benches.
IV. Some roads are houses.
A. Only I and II follow
B. Only I and III follow
C. Only III and IV follow
D. Only II, III and IV follow
E. None of these

15. Statements :
Some sticks are lamps.
Some flowers are lamps.
Some lamps are dresses.
All dresses are shirts.
Conclusions :
I. Some shirts are sticks
II. Some shirts are flowers
III. Some flowers are sticks
IV. Some dresses are sticks.
A. None of follows
B. Only I follows
C. Only II follows
D. Only III follows
E. Only IV follows

Directions (Qs. 16-20) : Study the following information carefully and answer the questions
given below;
A, B, C, D, E, F, G and H are eight employees of an organization working in three
departments viz, personnel, Administration and Marketing with not more than three of them
in any department. Each of them has a different choice of sports from football, cricket,
volleyball, badminton, lawn tennis, basketball, hockey and table tennis None of these
necessarily in the same order.
D works in Administration and does not like either football or cricket. F works in Personnel
with only A who likes table tennis. E and H do not work in the same department as D. C
likes hockey and does not work in Marketing. G does not work in Administration and does
not like either cricket or badminton. One of those who work in Administration likes football.
The one who likes volleyball works in Personnel. None of those who work in Administration
likes either badminton or lawn tennis. H does not like cricket.
16. Which of the following groups of employees work in Administration department?
A. EGH B. AF C. BCD D. BGD E. Data inadequate

17. In which department does E work ?


A. Personnel
B. Marketing
C. Administration
D. Data inadequate
E. None of these

18. Which of the following combinations of employee department-favourite sport is correct


?
A. E- Administration Cricket
B. F-Personnel-Lawn Tennis
C. H-Marketing-Lawn Tennis
D. B-Administration Table Tennis
E. None of these

19. What is Es favourite sport ?


A. Cricket B. Badminton C. Basketball D. Lawn Tennis E. None of these

20. What is Gs favourite sport ?


A. Cricket B. Badminton C. Basketball D. Lawn Tennis E. None of these

IBPS Bank PO CWE Symbols and Notations Blood Relationship Questions


Directions (Qs. 21-25) : In the following questions, the symbols @, $, *, # and & are used
with the following meaning as illustrated below;
P $ Q means P is not smaller than Q.
P @ Q means P is neither smaller than nor equal to Q
P # Q means P is neither greater than nor equal to Q.
P& Q means P is neither greater than nor smaller than Q.
P 8 Q means P is not greater than Q.
Now in each of the following questions assuming the given statements to be true, find which
of the four conclusions I, II, III and IV given below them is/are definitely true and give your
answer accordingly.
21. Statements : H @T, T#F, F&E, E*V
Conclusions : I. V $F
II. E @ T
III . H @ V
IV. T # V
A. Only I, II and III are true
B. Only I, II and IV are true
C. Only II, III and IV are true
D. Only I, III and IV are true
E. All I, II, III and IV are true.

22. Statements : D # R, R * K, K @ F, F $ J
Conclusions : I. J # R

II. J # K
III. R # F
IV. K @ D
A. Only I, II and III are true
B. Only II, III and IV are true.
C. Only I, III and IV are true.
D. All I, II, III and IV are true.
E. None of these.

23. Statements : N & B, B $ W, W # H, H * M


Conclusions : I. M @ W
II. H @ N
III. W & N
IV. W # N
A. Only I is true
B. Only III is true.
C. Only IV is true.
D. Only either III or IV true.
E. Only either III or IV and I are true.

24. Statements : R * D , D $ J, J #M, M @ K


Conclusions : I. K # J
II. D @ M
III. R # M
IV. D @ K
A. None is true
B. Only I is true
C. Only II is true
D. Only III is true
E. Only IV is true.

25.Statements : M $ K, K @ N, N * R, R #W
Conclusion : I. W @ K
II. M $ R
III. K @ W
IV. M @ N
A. Only I and II are true.
B. Only I, II and III are true.
C. Only III and IV are true.

D. Only II, III and IV are true.


E. None of these.

Directions (Qs. 26-30) : Study the following information carefully and answer the questions
given below :
Following are the conditions for selecting Senior Manager-Credit in a bank.
The candidate must -------------(i) be a graduate in any discipline with atleast 60 per cent marks.
(ii) Have post qualification work experience of at least ten years in the Advances Section of
a bank.
(iii) Be at least 30 years and not more than 40 years as on 01.04.2010.
(iv) Have secured at least 40 per cent marks in the group of discussion.
(v) Have secured at least 50 per cent marks in interview.
In the case of a candidate who satisfies all the conditions EXCEPT________
I. at (i) above but has secured at least 50 per cent marks in graduation and at least 60
percent marks in post graduation in any discipline the case is to be referred to the General
Manger-Advance.
II. at (ii) above but has total post qualification work experience of at least seven years out
of which at least three years as Manager-Credit in a bank, the case is to be referred to
Executive Director.
In each question below details of one candidate is given. You have to take one of the
following courses of action based on the information provided and the conditions and subconditions given above and mark the number of that course of action as your answer. You
are not to assume anything other than the information provided in each question. All these
cases are given to you as on 01.04.2010.
Give answer (A) if the case is to be referred to Executive Director.
Give answer (B) if the case is to be referred to General Manager-Advances.
Give answer (C) if the data are in adequate to take a decision.
Give answer (D) if the candidate is not to be selected.
Give answer (E) if the candidate is to be selected.

26. Shobaha Gupta has secured 50 per cent marks in the Interview and 40 per cent marks
in the Group Discussion. She has been working for the past eight years out of which four
years as Manager-Credit in a bank after completing her B.A degree with 60 per cent marks.
She was born on 12th September 1978.

27. Rohan Maskare was born on 8th March 1974. He has been working in a bank for the
past twelve years after completing his B.Com. degree with 70 per cent marks. He has
secured 50 per cent marks in both the Group Discussion and the Interview.

28. Prakash Gokhale was born on 4th August 1977. He has secured 65 per cent marks in
post graduation and 58 per cent marks in graduation. He has been working for the past ten
years in the Advances Department of a Bank after completion his post graduation. He has
secured 45 per cent marks in the Group Discussion and 50 per cent marks in the Interview.

29. Sudha Mehrotra has been working in the Advance department of a bank for the past
twelve years after completing her B. Com. Degree with 60 per cent marks. She has secured
50 per cent marks in the Group Discussion and 40 per cent marks in the Interview. She was
born on 15th February, 1972.

30. Amit Narayan was born on 28th May 1974. He has been working in the Advances
department of a bank for the past eleven years after completing his B.Sc degree with 65
per cent marks. He has secured 55 per cent marks in the Group discussion and 50 per cent
marks in the interview.

Directions (Qs 31-35) : In each question below is given a statement followed by three
courses of action numbered (1), (2) and (3). A course of action is a step or administrative
decision to be taken for improvement, follow-up or further action in regard to the problem,
policy, etc. On the basis of the information given in the statement, you have to assume
everything in the statement to be true, then decide which of the suggested courses of action
logically follow(s) for pursuing.

31. Statement :
A heavy unseasonal downpour during the last two days has paralysed the normal life in the
state in which five person were killed but this has provided a huge relief to the problem of
acute water crisis in the state.
Courses of Action :
(1) The state government should set up a committee to review the alarming situation.
(2) The state government should immediately remove all the restrictions on use of potable
water in all the major cities in the state.
(3) The state government should send relief supplies to all the affected areas in the state.
A. None
B. Only (1)
C. Only (2) and (3)

D. Only (3)
E. All (1), (2), (3)
32. Statement :
A large private bank has decided to retrench one-third of its employee in view of the huge
losses incurred by it during the past three quarters.
Course of Action :
a. The Govt. should issue a notification to general public to immediately stop all
trancsactions with the bank.
b. The Govt. should direct the bank to refrain from retrenching its employees.
c. The Govt. should ask the central bank of the country to initiate an enquiry into the banks
activities and submit its report.
A. None
B. Only (1)
C. Only (2)
D. Only (3)
E. Only (1) and (3)

33. Statement :
Many political activists have decided to stage demonstrations and block traffic movement in
the city during peak hours to protest against the steep rise in prices of essential
commodities.
Course of Action :
(1) The Govt. should immediately ban all forms of agitations in the country.
(2) The police authority of the city should deploy additional forces all over the city to help
traffic movement in the city.
(3) The State administration should carry out preventive arrests of the known criminals
staying in the city.
A. Only (1)
B. Only (2)
C. Only (3)
D. Only (1) and (2)
E. None of these.

34. Statement :

The school dropout rate in many districts in the state has increased sharply during the last
few years as the parents of these children make them work in the field owned by others to
earn enough for them to get at least one meal a day.
Course of Action :
(1) The Govt. should put up a mechanism to provide food grains to the poor people in these
districts through public distribution system to encourage the parents to send their wards to
school.
(2) The Govt. should close down some of these schools in the district and deploy the
teachers of these schools to nearby schools and also ask remaining students to join these
schools.
(3) Govt. should issue arrest warrants for all the parents who force their children to work in
fields instead of attending classes.
A. Only (1)
B. Only (2)
C. Only (3)
D. Only (1) and (2)
E. None of these

35. Statement :
One aspirant was killed due to stampede while participating in a recruitment drive of police
constables.
Course of Action :
(1) Official incharge of the recruitment process should immediately be suspended.
(2) A team of officials should be asked to find out the circumstances which led to the death
of the aspirant and submit its report within a week.
(3) The Govt. should ask the home department to stagger the number of aspirants over
more number of days to avoid such incidents in future.
A. Only (1)
B. Only (2)
C. Only (3)
D. Only (2) and (3)
E. None of these

36. Effect : Majority of the employees of the ailing organization opted for voluntary
retirement scheme and left the organization with all their retirement benefits within a
fortnight of launching the scheme.
Which of the following can be a probable cause of the above effect ?

A. The company has been making huge losses for the past five years and is unable to pay
salary to its employees in time.
B. The management of the company made huge personal gains through unlawful activities.
C. One of the competitors of the company went bankrupt last year.
D. The Company owns large tracts of land in the state which will fetch huge sum to its
owners.
E. None of these.

37. Statement : Most of the companies in IT and ITES sectors in India have started hiring
from engineering college campuses this year and are likely to recruit much more than yearly
recruitment of the earlier years.
Which of the following substantiates the facts stated in the above statement ?
A. IT and ITES are the only sectors in India which are hiring from engineering college
campuses.
B. Govt. has stepped up recruitment activities after a gap of five years.
C. The IT and ITES companies have now decided to visit the engineering college campuses
for tier II cities in India as well.
D. Availability of qualified engineers will substantially increase in the near future.
E. None of these

38. Cause : The Govt. has recently increased its taxes on petrol and diesel by about 10 per
cent.
Which of the following can be a possible effect of the above cause ?
A. The Petroleum companies will reduce the prices of petrol and diesel by about 10 per cent.
B. The petroleum companies will increase the prices of petrol and diesel by about 10 per
cent .
C. The petroleum companies will increase the prices of petrol and diesel by about 5 per
cent.
D. The petroleum pumps will stop selling petrol and diesel till the taxes are rolled back by
the govt.
E. None of these.

39. Statement : The Govt. has decided to instruct the banks to open new branches in such a
way that there is one branch of any of the banks in every village of population 1000 and
above or a cluster of villages with population less than 1000 to provide banking services to
all the citizens.

Which of the following will weaken the step taken by the Govt. ?
A. The Private sector banks in India have stepped up their branch expansion activities in
rural India.
B. Many Govt. owned banks have surplus manpower in it urban brances.
C. All the banks including those in private sector will follow the govt. directive.
D. Large number of branches of many Govt. owned banks in the rural areas are making
huge losses every year due to lack of adequate business activities.
E. None of these.

Directions (Qs. 40-42) : Study the following information carefully and answer the questions
given below;
The center reportedly wants to continue providing subsidy to consumers for cooking gas and
kerosene for five more years. This is not good news from the point of view of reining in the
fiscal deficit. Mounting subventions for subsidies means diversion of savings by the
government from investment to consumption, raising the cost of capital in the process. The
government must cut expenditure on subsidies to create more fiscal space for investments
in both physical and social infrastructure. It should outline a plan for comprehensive reform
in major subsidies including petroleum, food and fertilizers and set goal posts.
40. Which of the following is a conclusion which can be drawn from the facts stated in the
above paragraph?
A. Subsidy provided by the government under various heads to the citizen increases the
cost of capital.
B. Govt. is unable to withdraw subsidies provided to various items.
C. Govt. subsidy on kerosene is purely a political decision.
D. Govt. does not have enough resources to continue providing subsidy on petroleum
products.
E. None of these.

41. Which of the following is an inference which can be made from the facts stated in the
above paragraph ?
A. Indias fiscal deficit is negligible in comparison to other emerging economies in the world.
B. Subsidy on food and fertilizers are essential for growth of Indian economy.
C. Reform in financial sector will weaken Indias position in the international arena.
D. Gradual withdrawal of subsidy is essential for effectively managing fiscal deficit in India.

E. None of these.

42. Which of the following is an assumption which is implicit in the facts stated in the above
paragraph ?
A. People in India may not be able to pay more for petroleum products.
B. Many people in India are rich enough to buy petroleum products at market cost.
C. Govt. may not be able to create more infrastructural facilities if the present level of
subsidy continues for a longer time.
D. Govt. of India has sought assistance from International financial organizations for its
infrastructural projects.
E. None of these.

Directions (Qs. 43-45) : Study the following information carefully and answer the questions
given below;
Poverty measurement in an unsettled issue, both conceptually and methodologically. Since
poverty is a process as well as an outcome; many come out of it while others may be falling
into it. The net effect of these two parallel processes is a proportion commonly identified as
the head count ratio, but these ratios hide the fundamental dynamism that characterizes
poverty in practice. The most recent poverty reestimates by an expert group has also
missed that crucial dynamism. In a study conducted on 13,000 house holds which
represented the entire country in 1993-94 and again on 2004-05, it was found that in the
ten-year period 18.2% rural population moved out of poverty whereas another 22.1 % fell
into it over this period. This net increase of about four percentage point was seen to have a
considerable variation across states and regions.

43. Which of the following is conclusion which can be drawn from the facts stated in the
above paragraph ?
A. Accurate estimates of number of people living below poverty line in India is possible to be
made.
B. Many expert groups in India are not interested to measure poverty objectively.
C. Process of poverty measurement needs to take into account various factors to tackle its
dynamic nature.
D. People living below poverty line remain in that position for a very long time.
E. None of these.

44. Which of the following is an assumption which is implicit in the stated in the above
paragraph ?

A. It may not be possible to have an accurate poverty measurement in India.


B. Level of poverty in India is static over the years.
C. Researchers avoid making conclusions on poverty measurement data in India.
D. Govt. of India has a mechanism to measure level of poverty effectively and accurately.
E. None of these.

45. Which of the following is an inference which can be made from the facts stated in the
above Paragraph ?
A. Poverty measurement tools in India are outdated.
B. Increase in number of person falling into poverty varies considerably across the country
over a period of time.
C. Govt. of India has stopped measuring poverty related studies.
D. People living in rural areas are more susceptible to fall into poverty over the time.
E. None of these.
IBPS Bank PO CWE Non Verbal Reasoning Questions
Select a figure from amongst the Answer Figures which will continue the same series as
established by the five Problem Figures.

Ans C In each step, element at the upper-right position gets enlarged, inverts vertically
and reaches the lower-left corner; the existing element at the lower-left position, is lost and
a new small element appears at the upper-right position.
One, two, three, one, two, three.....arcs get inverted sequentially. This inversion takes place
in an ACW direction.

Complete preparation kit for Bank P O and Common Written Examination


IBPS Reasoning and aptitude Paper 5345
IBPS previous question papers
APTITUDE SECTION
1.Find the average of 38,64,27,76 & 40
a) 54
b)49
c) 78
d)45
ans: b
2.Find the average of 8,16,9,24,36,7,71 &37
a)62
b)46
c)36
d)52
ans: c
3.Find the average of 1 ,2 , &3 2/5
a)147/30
b)119/30 c)151/60 d)51/20
ans: d

4.find the average of 2/3,2 .,3 & 4 .


a)2 25/48
b)4 3/16 c)2 17/24
d)2 35/48
ans: d
5.Average of quantities is 22 and average of 36 quantities is 25. What is the average of all
the 50 quantities?
a)28.32 b)23.16 c)24.16
d)22.08
ans:c
6.The average of 42 observations is 36 and average of 18 observations is 24. What is the a
average of all the 60 observations?
a)28.5 b)37.6 c)32.4 d)21.52
ans: c
7.Average marks of a candidate in 6 subjects are 52.His marks in 5 subjects are
6o,48,36,55 and 51.Find his marks in the 6th subject.
a)12
b)48 c)62
d)data inadequate
ans:c
8)The average marks of five friends in English are 61.Four of them got 48,72,59 &38.Find
the marks of the fifth friend ?
a)18 b)88 c)84
d)66
ans:b
9. The average marks obtained by Raghu in Hindi & science were 30 less than his marks in
Hindi .he got 62 marks in science .Find his marks in Hindi?
a)122 b)120
c)124
d)102
ans:a
10.The average marks obtained by Ramesh in hindi ,English science were 15 more than his
marks in English .he got a total of 102 marks in hindi &science .How many marks did he get
in English?
a)26.5 b)29
c)30.5 d)28.5
ans:d
11.what percent of 72 is 6?
a)8 1/3 b)3 1/3
c)6 1/3 d)9 1/3
ans:a
12.What percent of 40 minutes is 24 seconds ?
a)1 b)10 c)100 d)0.1
ans:a
13.what percent of 20 quintals is 100 kgs?
a)2 b)0.5
c)20
d)5
ans:d
14.What percent is 45 praise of rs.90?
a)0.5
b)2 c)10
d)20
15.What percent is 40 kgs of 2 tones?
a)2 b)20
c)10
d)25
ans:a
16.As salary is 50% more than Bs ,How much percent is Bs salary less thanAs?
a)33 %
b)33 1/3%
c)33 %
d)33%
ans:b
17.If the are of a son is 25% less than the age of his father ,how much percent is the age of

father more than that of his son?


a)25%
b)50%
c)33 1/3%
d)50%
ans:c
18.The salary of A is 10% more than the salary of B .By what percent is the salary of B
lessthan that of A?
a)10%
b)1% c)11 1/9%
d)9 1/11%
ans:d
19.If As income is 40% less than that of B,how much percent Bs income is more than that
of A?
A)60% B)40%
C)66.66%
D)33.33%
Ans:c
20.The price of an article is increased by 20% & afterwards it is decreased by 20% find the
overll% change .a)40%
b)no change
c)4%dec
d)4% inc
ans:c
21.Find the simple interest on Rs.7,500 in 4 years at 15% per annum?
a)Rs.2,750 b)Rs.2,500 c)Rs.4,500
d)Rs. 5,000
ans:c
22.Find the s.i on Rs.8,000 in 3 years 12% per annum?
a)Rs.2,760 b)Rs3,760 c)Rs.4,250 d)Rs.3,360
ans:d
23.On what sum of money will be the S.I be Rs.2,000 in 5 years at 8% per annum?
a)Rs.4,500 b)Rs.5,000 c)Rs.5,200 d)Rs.5,300
ans:b
24. On what sum if money will the S.I be Rs.2,400 in 2 year at 12% per annum?
a)Rs.8,000 b)Rs.6,000 c)Rs.7,500
d)Rs.8,500
ans:a
25.The S.I on Rs.6,000 in 3 years and 4 months is Rs.3,00 Find the r% per annum?
a)10 b)12 c)15 d)18
ans:c
Reasoning
1.If TAP is coded as SZO ,then how is FREEZE coded?
a)EQDFYG
b)ESDFYF
c)GQFDYF
d)EQDDYD
ans:d
2.In a certain code. SIKKIM is written as THLJJL .How is TRAINING written in that code?
a)SQBHOHOH
b)UQBHOHOF
c)UQBJOHHO
d)UQBJOHOH
ans:b
3.In a certain code, MENTION is written as LNEITNO. How is PATTERN written in that
code?
a)APTTREM
b)PTAETNR
c)OTAETNR
d)OTAETRN
ans:c
4.In a certain code, FORGE is written as FPTJI .How is CULPRIT written in that code?
a)CSJNPGR
b)CVMQSTU
c)CVNSVNZ
d)CXOSULW
ans:c
5.If in a code , ALTERED is written as ZOGNINW, then in the same code , RELATED would be
written ?
a)IVOZGVW
b)IVOZGWV
c)IVOGZVW
d)VIOZGVW

ans:a
If MISTAKE is coded as 9765412 and NAKED is coded as 84123 , how are the following
words coded ?
6.DISTANT
a)3765485
b)4798165
c)3697185
d)4768296
7.NEMISES
a)7598656
b)8597656
c)8297626
d)7689565
8.ASSIST
a)166762
b)466765
c)488976
d)435985
9.INTIMATE
a)89786145
b)79438163
c)78579452
d)78698365
10.STAIN
a)98175
b)89483
c)68194
d)65478
6.ans:a
7. ans:c
8.ans:b
9. ans:c
10. ans:d
LETTER ANALOGY
11.AB:ZY:CD:?
a)XY
b)EF
c)XW
d)AB
ans:c
12.AEGK:MQ:?
a)ST
b)SW
c)UV
d)WS
ans:b
13.BC:GH:ST:?
a)YZ
b)UV
c)XY
d)WX
ans:c
14.PS:DG:QT:?
a)EH
b)GH
c)FF
d)EG
ans:a
15.V:X:G?
a)Y b)T
c)I
d)J
ans:c
16.GT:HS:KP:?
a)PQ
b)RS
c)LO
d)LM
ans:c
17.CDF:GHJ:KLN:?
a)ORP
b)PQR
c)OPR
d)RSU
ans:c
18.ABC:CBA:DEF:?
a)FED
b)EFD
c)DEF
d)GHI
ans:a
19.ZWT:SM:LIF:?
a)EBY
b)EAZ
c)EBX
d)EAX
ans:a
20.ABZ:CDY:EFW:?
a)FGU
b)VHG
c)GHK
d)GHV
ans:d
LETTER SERIES

21.U,O,I,?,A
a)E
b)C
c)S
d)G
ans:a
22.A,B,D,G,?
a)M
b)L
c)k
d)h
ans:c
23:Z,U,Q,?,L
a)I
b)K
c)M
d)N
ans:c
24.A,C,F,H,?,M
a)L
b)K
c)J
d)I
ans:b
25.Z,L,X,J,V,H,T,F,?,?
a)R,D
b)R,E
c)S,E
d)Q,D
ans:a
26.Z,S,W,O,T,K,Q,G,?
a)N,C
b)N,D
c)O,C
d)O,D
ans:a
27.W,V,T,S,Q,P,N,M,?,?
a)I,J
b)J,I
c)J,K
d)K,J
ans:d
REPEATED LETTER SERIES
1_ _aba_ _ ba_ab
a)abbba
b)bbab
c)baabb
d)bbaba
ans:b
2.ab _ _ _ b _bbaa
a)abaab
b)abbab
c)baaab
d)babba
ans:c
3._baa _aab _a _a
a)aabb
b)aaba
c)abab
d)baab
ans:c
4_ _ babbba _a_ _
a)ababb b)baaab
c)bbaba d)babbb
ans:d
5.aa_ ab_ aaa _a
a)aaab b)aabb c)abab d)baaa
ans:a
6.a _bbc _aab _cca _bbcc_
a)bacb b)acba c)abba
d)caba
ans:b
7ab _aa _bbb _aaa _bbba _
a)abba
b)baab c)aaab d)abab
ans:b
8.bc _b _c _b _ ccb_
a)cbcb b)bbcb c)cbbc
d)bcbc
ans:a

9.abb _baa _a _bab _aba


a)abba
b)abab
c)ccac
d)aabb
ans:a
10.abca _bcaab _ca _bbc _ a
a)ccaa b)bbaa
c)abac
d)abba
ans:c

You might also like